Family Med Aquifer Questions

Lakukan tugas rumah & ujian kamu dengan baik sekarang menggunakan Quizwiz!

Which of the following is correct regarding breast self-examination? Choose the single best answer. A. Breast self-examination increases the number of biopsies performed. B. It is recommended that clinicians teach breast self-examination during annual visits. C. Most cisgender women should regularly perform breast self-examination. D. The practice of regular breast self-examination reduces mortality.

A

Would you recommend a medication to a patient with stage 2 HTN? If so, which medication? Select all that apply. A. Yes; initiate a beta-blocker B. Yes; initiate a calcium-channel blocker C. Yes; initiate a thiazide diuretic D. Yes; initiate an ACE-Inhibitor E. No antihypertensive medication indicated at this time SUBMIT

B, C, D

A 24-year-old G1P0 patient at 38 weeks gestation presents to ED complaining of strong lower abdominal contractions that are 10 minutes apart for the last hour. Subsequent cervical examination demonstrates that she is 2cm dilated. FHT is 140 and NST is non-reactive with early decelerations. What is the most appropriate management of this patient? A. Augment labor with oxytocin (Pitocin) B. C-section C. Expectant management D. Immediate vacuum delivery E. Rupture membranes to increase labor

C. Expectant management

A 28-year-old, G2P1 patient delivers a 6 lb., 7oz. baby boy at 39 weeks gestation. At one minute, the baby has blue extremities and a pink body; his arms and legs are flexed and he is moving them vigorously with prompt response to stimulation; HR is 118 bpm and he is coughing and crying vigorously as well. What would his APGAR score be at one minute? A. 6 B. 7 C. 8 D. 9 E. 10

D. 9

Which treatments are recommended in the elderly? The best option is indicated below. Your selections are indicated by the shaded boxes. A. Antidepressants B. Antihistamines C. Benzodiazepines D. Cognitive behavioral therapy E. Zolpidem

D. Cognitive behavioral therapy

Which of the following are characteristics of a good screening test? Select all that apply. A. There should be a latent (asymptomatic) stage of the disease. B. There should be a treatment for the condition being screened for. C. The test should be acceptable to the population. D. The test should have low sensitivity and high specificity. E. The total cost of finding a case should be economically balanced in relation to medical expenditure as a whole. SUBMIT

The correct answers are A, B, C, E.

"What if he presented with one week of upper respiratory symptoms including nasal congestion and drainage? On the day the symptoms began, he had a low-grade fever that has now resolved. His nasal congestion persisted and he has now had yellow nasal drainage for five days with associated mild headaches. On exam, he has a small amount of yellowish-clear drainage. There is tenderness when you lightly percuss his maxillary sinuses." What would your treatment plan be for the patient in this situation? Choose the single best answer. A. Observation and reassurance B. Treatment with an antibiotic such as amoxicillin C. Treatment with an antibiotic such as a fluoroquinolone

A The patient in the example above is presenting with symptoms consistent with acute viral rhinosinusitis (the common cold). In the setting of a viral infection, prescribing antibiotics (B or C) would be inappropriate.

Which of the following are true about depression among minority populations in the U.S.? Select all that apply. A. Asian, Black and Hispanic patients with depression are less likely than whites to perceive a need for mental health treatment. B. Hispanic patients are less likely to be diagnosed than non-Hispanic whites. C. Hispanics have higher rates of depression than non-Hispanic whites. D. Minority populations are more likely to suffer from psychotic depression. E. Racial, ethnic and economic minorities receive evidence-based treatments at the same rates as White Americans. SUBMIT

A and B. U.S.-born Hispanics experience depression at similar rates to other ethnic groups. Rates of depression in immigrant Hispanics are up to 50 percent lower than U.S.-born Hispanics. Psychotic features in depression are no more common in minority populations than non-Hispanic whites.

As you step out of the room, you realize that you just gave Chantel a lot of information. It's hard for patients to remember everything that they hear in a doctor's visit. Which of the following should you do to ensure the patient and family understand the information you have related? Select all that apply. A. Give her a handout with the appropriate anticipatory guidance. B. Give her the name of some books that have the information in it. C. Have the nurse review the guidance points with her. D. Have the patient repeat back the information so that you know what she understands and remembers. E. Refer her to a website that has all the information that she needs.

A, B, C, D, E

Besides amenorrhea, which of the following are signs and symptoms of pregnancy? Select all that apply. A. Breast tenderness B. Cervix and vaginal walls have an increasingly purplish-blue hue C. Enlargement of the uterus D. Fatigue E. Fetal heart tones F. Fetal movement G. Nausea H. Softening of the cervix and uterus I. Urinary frequency J. Vomiting SUBMIT

A, B, C, D, E, F, G, H, I, J. Urinary frequency can also occur. Although urinary frequency can be a normal symptom of pregnancy, the possibility of a UTI should also be considered. Softening of the cervix is known as Goodell's sign, while softening of the uterus is known as Hegar's sign. The bluish-purple hue in the cervix and vaginal walls is known as Chadwick's sign and is caused by hyperemia. Enlargement of the uterus can be detected by an experienced examiner as early as 8 weeks on bimanual exam. Around 12 weeks, the uterine fundus can be palpated above the symphysis pubis. Between 20 to 36 weeks of gestation, the uterine enlargement, measured in centimeters, approximates gestational age and will become a routinely elicited physical exam finding. Fetal heart tones are typically elicited by hand-held Doppler between 10-12 weeks gestation. Fetal movement or "quickening" is detected by the mother around 18-20 weeks of gestation.

What history related to Mr. Reynolds' risk of cardiovascular disease do you want to obtain? Select all that apply. A. Excess alcohol use B. Exercise C. Family history D. History of chest pain with exercise E. History of leg pain with exercise F. Stress G. Tobacco use H. Travel history

A, B, C, D, E, F, G.

Although Savannah is not planning on getting pregnant, you have informed her about folic acid supplements because any sexually active woman should be aware of preconception counseling in case she unintentionally becomes pregnant. What other issues would you want to address with women who are trying to get pregnant? Select all that apply: A. Assessing and advising against alcohol and tobacco use B. Assessing current weight and nutritional status C. Family history of cystic fibrosis D. Reviewing current medications E. Risk factors for HIV F. Screening for safety and domestic violence SUBMIT

A, B, C, D, E, F.

What are the risk factors for developing breast cancer in the general population? Select all that apply. The best options are indicated below. Your selections are indicated by the shaded boxes. A. Age B. Excessive alcohol intake C. Family history of breast cancer in first-degree relative D. Genetic factors E. Postmenopausal obesity F. Prolonged exposure to estrogen

A, B, C, D, E, F.

Which of the following are long-term effects of asthma? Select all that apply. A. Airway remodeling B. Airway smooth muscle hypertrophy C. Angiogenesis D. Incomplete reversibility of airway obstruction E. Mucous gland hyperplasia and hypersecretion F. Subepithelial fibrosis

A, B, C, D, E, F.

Which of the following can cause wheezing? Select all that apply. A. Asthma B. Chronic obstructive pulmonary disease C. Congestive heart failure D. Epiglottitis E. Foreign body aspiration F. Obstructive sleep apnea G. Persistent bronchitis H. Pneumonia I. Pulmonary embolism J. Upper airway cough syndrome K. Vocal cord dysfunction

A, B, C, D, E, G, H, I, J, K

Dr. Medel asks you, "What additional tests would be appropriate to order at this time for a pregnant pt?" Select all that apply. A. Blood type/Rh status B. Chlamydia screening (urine or swab) C. Hemoglobin/hematocrit D. Hepatitis B surface antigen E. Hepatitis C antibody F. Herpes I/II antibody G. HIV test H. Screening for bacterial vaginosis I. Toxoplasmosis J. Ultrasound (for dating)

A, B, C, D, E, G. - Herpes I/II antibody (F): Type-specific HSV serology may be appropriate in some patients with a history of HSV, but not in a female with no prior history of herpes. - Bacterial vaginosis (H): Screening for bacterial vaginosis is not recommended. - Toxoplasmosis (I): Universal screening for toxoplasmosis, cytomegalovirus, and parvovirus is not recommended. - Ultrasound (J): Since Ms. Rios has reliable LMP dates, a screening ultrasound would not be necessary at this time.

When Savannah has left, Dr. Hill asks you, "What lab tests would be ordered as part of the initial pregnancy workup plan?" Select all that apply. A. Complete blood count B. Hepatitis B surface antigen C. HIV D. RPR E. Rubella antibodies F. Serum pregnancy test G. Type & screen H. Ultrasound

A, B, C, D, E, G. -An ultrasound would not be the best test to order at an early stage of pregnancy. For example, at five weeks' estimated gestation, an embryo would typically not be seen. Furthermore, the results would be difficult to interpret without a serum quantitative beta-human chorionic gonadotropin test (quantitative pregnancy test).

Which of the following are triggers that may increase the frequency and severity of Sarah's headaches? Select all that apply. A. Aspartame and phenylalanine B. Caffeine C. Emotional stress D. Intense exercise E. Sleep disturbance

A, B, C, D, E.

Which of the following are true regarding the natural history of H. pylori infection? Select all that apply. A. H. pylori is spread through human saliva and feces and via food and water sources. B. Ninety percent of patients worldwide with duodenal ulcers are infected with H. pylori. C. The gastric mucosa offers H. pylori protection from host immune mechanisms. D. The prevalence of H. pylori is declining worldwide. E. The strongest evidence to support the role of H. pylori as an etiology of PUD is the elimination of ulcer recurrence after eradication.

A, B, C, D, E.

In addition to the pelvic ultrasound, which of the following laboratory tests would be reasonable to obtain at this juncture to Investigate First Trimester Vaginal Bleeding? Select all that apply. A. Complete blood count (CBC) B. PCR for gonorrhea and chlamydia C. Progesterone D. Quantitative beta-human chorionic gonadotropin (quant. beta-hCG) E. Type and screen (Rhesus typing) F. Wet mount preparation

A, B, C, D, F.

Which of the following are in your differential for vaginal bleeding in a post-menopausal woman on hormone replacement? Select all that apply. A. Cervical polyp B. Endometrial cancer C. Endometrial hyperplasia D. Hormone-producing ovarian tumors E. Normal bleeding with hormone replacement and no further workup warranted F. Proliferative endometrium

A, B, C, D, F. Normal bleeding with hormone replacement (E) is incorrect because this patient has been on daily HRT for more than 12 months. Bleeding in the first 12 months of HRT can be normal, however, bleeding after 12 months of therapy always requires further evaluation.

What do you think are the risk factors for developing cervical cancer? A. Cigarette smoking B. Early onset of sexual intercourse C. Immunosuppression from HIV or other diseases D. Multiple sexual partners

A, B, C, D.

What strategies can be used to prevent osteoporosis? Select all that apply. A. Adequate intake of Vitamin D B. Dietary Calcium intake of 1200 mg per day C. Smoking cessation D. Weight bearing exercise

A, B, C, D.

Which of the following are appropriate next steps to offer Mr. Rodriguez at this time after his epigastric pain has resolved following treatment for H. pylori Select all that apply. A. Advisement to continue to minimize alcohol consumption B. A follow-up appointment if symptoms of dyspepsia or any new symptoms recur C. A follow-up appointment for a health maintenance examination D. Positive reinforcement regarding smoking cessation

A, B, C, D.

Which of the following are treatment options for osteoporosis? Select all that apply. A. Alendronate (Fosamax) B. Calcitonin C. Parathyroid Hormone Analogs (Forteo) D. Prednisone E. Raloxifene

A, B, C, E. These are all available treatments for osteoporosis except for prednisone (D). Prednisone and other corticosteroids actually increase the risk of osteoporosis when used long-term.

What are the management options for an inevitable abortion? Select all that apply. A. Expectant management B. Medical therapy C. Surgical management

A, B, C.

Which of the following are pain characteristics that argue against the likelihood of angina? Select all that apply. A. Palpation reproduced pain B. Pleuritic pain C. Positional pain D. Pressure

A, B, C.

"Which of the following findings would support the diagnosis of disc herniation?" Select all that apply. A. Drop foot B. Pain worse with cough and sneezing C. Pain worse with sitting D. Pain worse with standing E. Urinary retention

A, B, C. Urinary retention (E) is a symptom consistent with cauda equina syndrome. Standing typically relieves the symptoms of disc herniation (D), so it is incorrect here.

Which of the following statements are TRUE regarding acid-peptic disorders? Select all that apply. A. A diagnosis of GERD may be made with reasonable accuracy on the basis of classical symptoms and response to empiric acid suppression. B. Both peptic ulcer disease and functional dyspepsia may be associated with nausea and vomiting. C. Eating and drinking make GERD symptoms improve and PUD symptoms worse. D. GERD is distinctly different from dyspepsia E. Non-erosive reflux disease (NERD) is more common than erosive GERD. F. Patients with GERD report lower health-related quality of life than patients with heart failure.

A, B, D, E, F.

Which of the following would be appropriate to the evaluation of Mrs. Parker's post-menopausal bleeding? Select all that apply A. Complete blood count B. Endometrial biopsy C. Follicle-stimulating hormone (FSH) and luteinizing hormone (LH) levels D. Thyroid-stimulating hormone (TSH) level E. Transvaginal ultrasound

A, B, D, E. FSH and luteinizing hormone (LH) (C) would not be helpful in this case as Mrs. Parker has already met the criteria for being postmenopausal by the fact that she has not had any menses in more than a year.

What factors increase a patient's risk for completed suicide? Select all that apply. A. Having a previous suicide attempt. B. Having served in the military. C. Living in poverty. D. Male sex. E. Older age.

A, B, D, E. Poverty by itself has not been found to be a risk factor for completed suicide, though it can contribute to psychosocial stress and the development of depressive symptoms.

What are the red flags or alarm symptoms that would suggest a more serious underlying condition causing his back pain? Select all that apply. A. Fever B. Loss of bowel/bladder control C. Numbness of the leg D. Severe pain that awakens the patient from sleep. E. The worst pain the patient has ever had F. Weight loss

A, B, D, F. While the worst pain a patient has ever had is concerning and needs to be addressed, it is not by itself indicative of a more serious condition. Numbness can be part of cauda equina, but is also common with a simple disc herniation, therefore by itself it is not a red flag.

Dr. Nayar asks you, "What ECG changes would suggest that Mr. Reynolds has existing coronary artery disease?" Select all that apply. A. Convex ST segment elevation B. Q waves C. Short PR interval D. ST segment depression or downsloping ST segment E. U waves

A, B, D.

Which of the following statements are true regarding hyperosmolar hyperglycemic state (HHS)? Select all that apply. A. Dehydration is a common finding. B. Ketones are absent or mildly elevated. C. Metabolic acidosis is the primary disturbance. D. Plasma glucose levels are commonly > 600 mg/dL. E. The mortality rate is extremely low.

A, B, D.

Which pharmacologic and non-pharmacologic measures have roles in the treatment of Alzheimer dementia and its symptoms? Select all that apply. A. Atypical antipsychotics for behavioral disturbances B. Cholinesterase inhibitors C. Cognitive rehabilitation therapy D. Ginkgo biloba E. Memantine F. Respite care G. Vitamin E

A, B, E, F, G. -Cognitive rehabilitation therapy has not been demonstrated to be helpful in slowing the progression of Alzheimer dementia. -A 2007 Cochrane Collaboration systematic review concluded that the evidence for ginkgo biloba's role in the management of dementia is "inconsistent and unconvincing."

Which of the following are independent risk factors for coronary artery disease (CAD)? Select all that apply. A. Diabetes B. HDL cholesterol < 40 mg/dL C. History of premature coronary artery disease (CAD) in a second-degree relative. D. Moderate to intense exercise on the weekend only. E. Obesity F. Smoking

A, B, E, F.

Which of the following are necessary to evaluate when you suspect benign prostatic hyperplasia (BPH)? Select all that apply. A. Examination of prostate. B. Presence of classic lower urinary tract symptoms (LUTS). C. Residual volume of urine. D. Serum BUN and creatinine. E. Serum prostate specific antigen (PSA). F. Urinalysis. G. Urine flow rate.

A, B, E, F.

Which of the following features would suggest a diagnosis of Down syndrome? Select all that apply. A. Anomalous ears B. Flat facial profile C. Hypertonia D. Normal Moro reflex E. Slanted palpebral fissures F. Transverse palmar crease

A, B, E, F.

Which of the following interventions have evidence of improving cardiovascular disease outcomes that matter to patients with diabetes? Select all that apply. A. Adding a high-intensity statin for patients 40—75 yrs old with LDL-c > 70 mg/dL and ≥ 7.5% estimated 10-year ASCVD risk B. Adding a moderate-intensity statin for patients 40—75 yrs old with LDL-c > 70 mg/dl C. Advising all patients to cut back on their smoking D. Controlling glucose as close to normal range as possible (A1C 4 to 6%) E. Lowering blood pressure in patients with blood pressure > 140/90 mmHg F. Screening asymptomatic patients with diabetes for coronary heart disease (CHD) G. Treating patients with dyslipidemia with diet and exercise H. Using aspirin as secondary prevention in patients with diabetes and a history of CVD

A, B, E, G, H. Advising all patients to cut back on their smoking has not been shown to improve cardiovascular outcomes; rather, patients should be advised not to use tobacco products.

Which hospital interventions can prevent or minimize delirium? Select all that apply. A. Avoid sedative medications and medications with anticholinergic effects B. One-to-one sitter C. Place a Foley catheter to prevent the urge to urinate and incontinence D. Place patient in room at the end of the hall where it is dark and quiet at night E. Range of motion exercises F. Remove eyeglasses so the patient is not likely to pick at and damage them G. Repeated reorientation

A, B, E, G. Placing a patient at the end of the hall where it is dark and quiet at night would not be helpful. Mounting evidence suggests that sensory deprivation is a contributing factor to delirium in the hospital. Interventions that increase, rather than minimize, stimulation seem effective at preventing or minimizing delirium. As such, keeping patients in rooms with adequate lighting, windows, large clocks, and calendars helps keep the patient calm and more oriented. Keeping the patient close to the activity of the nursing station is also helpful. Visual and hearing disturbances can contribute to delirium. Dim lighting, ambient noise, and the removal of hearing and visual aids therefore should be avoided.

You recognize that Sarah is asking for a specific diagnostic test and wants it to be done urgently. Based on her history, you don't believe that Sarah will need an MRI urgently. Which of the following are the next best steps? Select all that apply. A. Expand your history and perform your physical examination. B. Identify and address her primary concerns. C. Tell her up front that she doesn't need an MRI at this time. D. Tell her you'll make sure she gets an MRI. E. Try not to bring up brain tumors.

A, B.

Which of the following options are appropriate next steps in evaluation and treatment with epigastric pain persisting after H. pylori treatment? Select all that apply. A. Obtain an H. pylori fecal antigen test B. Obtain a urea breath test C. Prescribe a PPI twice daily for 4 weeks, then reevaluate D. Refer for an upper endoscopy/EGD and biopsy E. Repeat a course of PPI triple therapy for 14 days F. Treat with salvage therapy for resistant H. pylori infection SUBMIT

A, B. -If the fecal antigen test or the urea breath test is positive, the patient will require re-treatment for a resistant infection. This should not be given prior to testing for the presence of active H. pylori infection. -If the fecal antigen test or urea breath test is negative, and the patient continues to have symptoms, he should be referred to a gastroenterologist for an upper endoscopy/EGD and mucosal biopsy.

What are the treatment options for radiculopathy of the S1 nerve root due to a large herniated disc at L5-S1? Select all that apply. A. Acupuncture B. Epidural injection C. It has only been five weeks, continue with current treatment D. Osteopathic manipulation E. Surgery

A, C, D, E.

Which of the following instruments may be used for screening pregnant and postpartum women for depression? Select all that apply. A. Edinburgh Postnatal Depression Scale B. GAD-7 C. PHQ-2 D. PHQ-9 E. C followed by D if positive

A, C, D, E. -The Edinburgh Postnatal Depression Scale (A) is a tool validated for use expressly to detect postpartum depression

Which of the following statements are true about the diagnosis of pregnancy? Select all that apply. A. Blood tests of human chorionic gonadotropin (hCG) can identify pregnancy as early as 8 days post-conception (about 1 week prior to a missed period). B. Home pregnancy tests are not a reliable indication of pregnancy. C. The presence of a delayed menstrual period in a female with previously regular periods and symptoms of nausea, breast tenderness, and fatigue are classical indications of pregnancy. D. Transvaginal ultrasound is a useful method for dating early pregnancy but is not typically used to diagnose pregnancy. SUBMIT

A, C, D.

"Now that you have a diagnosis of disc herniation with radiculopathy for Mr. Payne, let's discuss what would you like to do for him." Which of the following are indicated at this time? Select all that apply. A. Moist heat B. Order an MRI C. Prescribe NSAID D. Referral to a back surgeon E. Referral to physical therapy F. Strict bed rest

A, C, E. Mr. Payne has no red flags and has had pain for only two weeks, so conservative therapy is appropriate for him.

What are the most common causes of dementia? Choose the three best answers. A. Alzheimer disease B. Creutzfeldt-Jakob disease C. Dementia with Lewy bodies D. Huntington disease E. Parkinson disease F. Pick disease G. Vascular dementia

A, C, G

What is the first step in managing BPH? A. Behavior modifications to decrease symptoms B. Refer the patient to urology service for surgical intervention like TURP (transurethral resection of the prostate) C. Start with 5-alpha-reductase inhibitor D. Start with alpha-adrenergic antagonists E. Start with combination treatment of an alpha-adrenergic antagonist and a 5-alpha-reductase inhibitor

A, D Alpha-adrenergic antagonists decrease urinary symptoms in most males with mild to moderate BPH. Alpha-adrenergic antagonists include tamsulosin, alfuzosin, terazosin, and doxazosin. The American Urology Association (AUA) Guidelines Committee believes that all four medications are equally effective. 5-alpha-reductase inhibitors are more effective in males with larger prostates. Their effect on preventing acute urinary retention and reduction in need of surgery require long-term treatment for more than a year. There are two 5-alpha-reductase inhibitors approved in the United States: finasteride and dutasteride.

Which of the following are benefits of menopausal HT? Select all that apply. A. Decrease in vasomotor symptoms B. Decreased incidence of stroke C. Decreased risk of breast cancer D. Improvement in atrophic vaginitis symptoms E. Prevention of osteoporosis SUBMIT

A, D, E.

According to the National Asthma Education and Program guidelines, there is more than one option for maintenance medication to treat moderate persistent asthma. Which are recommended options to use as a maintenance medication for Mr. Dennison? Select all that apply. A. Combination of a low-dose inhaled corticosteroid and a long-acting beta2 agonist inhaler B. High-dose inhaled corticosteroid C. Leukotriene receptor antagonist D. Medium-dose inhaled corticosteroid E. Oral corticosteroid F. Theophylline

A, D.

After reviewing the treatment options in the preceding list, considering all aspects of Sarah's history and exam, which would be the best initial treatments for prevention of her migraine headaches if her lifestyle measures fail to reduce her symptoms to the acceptable level? Select all that apply. A. Amitriptyline B. Divalproex sodium C. Feverfew D. Propranolol E. Topiramate

A, D.

How would you interpret Savannah's vital signs and prenatal labs? Select all that apply. A. Rhesus immune globulin (RhoGam) is indicated. B. The low-grade fever suggests infection. C. The normal blood pressure suggests it is unlikely that she is bleeding now. D. The pulse is within normal limits, suggesting hemodynamic stability.

A, D.

What do you want to check as part of the back exam while the patient is supine? Select all that apply. A. Check for abdominal bruit, especially on older adult patients. B. Check for abdominal tenderness only in female patients. C. Perform FABER test only in older adult patients. D. Perform passive straight leg raise on all patients. E. Perform rectal exam on all patients.

A, D.

Which of the following are characteristics of migraine headaches? Select all that apply. A. Accompanied by nausea and vomiting. B. Alleviated by physical activity. C. Last 30 minutes to seven days. D. Moderate to severe pain. E. Pressing or tightening bilateral pain. F. Ten episodes are necessary for diagnosis. SUBMIT

A, D.

Among the following conditions, which need to be treated with systemic antifungal agents? Select all that apply. A. Tinea capitis B. Tinea corporis/tinea cruris C. Tinea pedis/tinea manuum D. Tinea unguium (onychomycosis)

A, D. - Oral therapy is required to adequately treat tinea capitis, as they are able to penetrate the infected hair shaft where topical therapies cannot

Next, she asks: "Which two of the following options are the most appropriate first steps in diagnostic testing and therapeutic planning for this patient?" Choose the two best answers. A. Focusing on lifestyle modifications to promote symptomatic improvement B. Ordering an upper GI series (barium swallow radiograph) C. Referring the patient for a 24-hour pH probe D. Referring the patient for an upper endoscopy (esophagogastroduodenoscopy/EGD) E. Using an empiric treatment strategy with a proton pump inhibitor (PPI)

A, E. An empiric treatment strategy for GERD, gastritis, FD, and PUD is the most widely accepted initial therapeutic intervention in patients without red flag symptoms.

Which bedside instruments used for diagnosing delirium does evidence best support? Choose the single best answer. A. Confusion Assessment Method (CAM) B. MiniCog C. Mini-Mental State Examination (MMSE)

A.

Which response to Mrs. Marshall's remark would be the most therapeutic? Choose the single best answer. A. "I agree with you that you haven't 'lost your marbles,' as you say. You're clearly doing a great job taking care of your husband. Was there anything she suggested that sounded as if it could be helpful to you?" B. "I'm sure she didn't mean to suggest a homemaker for you. You're clearly doing a great job taking care of your husband. I imagine she was talking about a home aide or a respite worker who would allow you to get a break from time to time." C. "It is hard for me to know what she meant to say, and home care services can be confusing. It will be helpful to have Dr. Wilson clarify things for you."

A. Answer A takes a non-confrontational stance by agreeing with her, and it offers Mrs. Marshall support for her care giving. This answer is also the most likely to elicit useful information from her, since it asks her a question. In this case, the question is asked in a closed-ended manner. It would be more effective to ask, "Which of the services she suggested to you sounded the most helpful?" Answer B makes a dangerous assumption—that you understand better what the nurse meant to say than Mrs. Marshall did. While this answer does offer Mrs. Marshall strong support for her caregiving, it is likely that she understood the nurse correctly. By implying otherwise, you take the risk of offending her and damaging your relationship.

Dr. Medel asks you, "If we are to follow the 2017 ACC/AHA hypertension guidelines, what is the goal blood pressure we want to achieve with this patient?" Choose the single best answer. A. < 130/80 mmHg B. < 140/80 mmHg C. < 150/90 mmHg

A. < 130/80 mmHg In the ACC/AHA guidelines, for all patients, the goal blood pressure is 130/80 mmHg.

A 17-year-old male football player presents to the emergency department with acute onset of right-shoulder pain. The pain started approximately two hours ago when he was tackled during a game while carrying the football. Shoulder swelling and pain were present after injury. He is holding ice to a swollen shoulder, and on exam, you notice severe bruising, pain on palpation over the lateral edge of the shoulder, and a raised bump over that same area. There is also severe tenderness over the coracoclavicular ligaments. What is the most likely diagnosis? A. AC joint sprain B. Rotator cuff tear C. Rotator cuff tendonitis D. Subacromial bursitis E. Subacromial impingement

A. AC joint sprain

A 62-year-old female presents for follow-up of her hypertension and diabetes. In general, these chronic diseases are well controlled and she has suffered no target organ damage. She has worked hard to begin exercising, and is walking vigorously five times a week. She has also worked hard on dietary changes, and has been following the DASH eating plan very seriously. She quit smoking three months ago. Her blood pressure today is 148/88 mmHg, pulse is 72 beats/minute, respiratory rate is 16 breaths/minute, and BMI is 32 kg/m2. She is taking metformin 500 mg twice daily, simvastatin 20 mg daily and hydrochlorothiazide (HCTZ) 25 mg daily, and she is adherent with her daily medications. Her labs today include an A1C of 6.6, an LDL of 88 and a basic metabolic panel within normal limits. Which of the following management steps today do you consider the most appropriate? A. Add amlodipine 5 mg daily B. Change her simvastatin from 20 mg to 40 mg C. Impress upon her the importance of making more lifestyle modifications D. Increase HCTZ to 50 mg daily E. Make no changes as she is at her treatment goals

A. Add amlodipine 5 mg daily

A 72-year-old female presents to the clinic one year after having suffered a stroke that left her with some residual left-sided weakness/paralysis. On exam, the left shoulder demonstrates a severe decrease in both active and passive ROM and significant pain. The patient has a history of hypertension, hyperlipidemia, and diabetes. What is the most likely diagnosis? A. Adhesive capsulitis B. Biceps tendonitis C. Osteoarthritis D. Rotator cuff tear E. Subacromial bursitis

A. Adhesive capsulitis

Ms. McSally is a generally healthy 27-year-old female graduate student who presented to your office twelve weeks ago with episodic postprandial epigastric burning. This had been bothering her for nearly six months but she had been busy with her thesis and was unable to find the time necessary for an appointment. She reported this year has been particularly stressful, with limited time resulting in increased consumption of coffee and takeout fast food. At that time, she began an eight-week trial of omeprazole. She returns now with no improvement of her symptoms. She discontinued the omeprazole one month ago because she ran out of the medication. She has no additional symptoms and physical exam is unremarkable. Which of the following is the most appropriate next step in her management? Choose the single best answer. A. Administer a urea breath test B. Begin treatment with triple therapy of pantoprazole, clarithromycin, and azithromycin for 14 days C. Continue omeprazole for another four weeks with close follow-up D. Refer her for an upper gastrointestinal endoscopy E. Switch PPI from omeprazole to pantoprazole

A. Administer a urea breath test This patient exhibits no alarm symptoms, but does have persistent symptoms despite adequate empiric therapy. This case warrants testing for H. pylori as a cause of symptoms. One test which is sensitive and specific for H. pylori infection is the urea breath test. The antibiotic regimen listed in choice B is for treatment of H. pylori, but a diagnosis must be made before instituting such a treatment regimen.

Ms. Michaels is an 80-year-old female with a past medical history of shingles. She comes to your office accompanied by her daughter Jennifer who reports that her mother is forgetting things. Jennifer explains that her mother will ask the same question several times throughout the day. Ms. Michaels also gets confused easily and is more passive than usual. Her memory problem was noticed two years ago after she forgot to pay her bills on multiple occasions. Jennifer now pays her mother's bills and cleans and cooks for her. Ms. Michaels' vital signs are: temperature of 37.3 C (99.2 F), blood pressure of 118/70 mmHg, pulse of 80 beats/minute, and respiratory rate of 12 breaths/minute. Her physical exam is significant for bilateral osteoarthritis hand deformities. She does not have a tremor, nor jerky uncontrolled movements. She is not on any chronic medications. She has no smoking history and does not drink alcohol. CT head shows mild atrophy of the hippocampus. Her MMSE is 20. Which of the following is the most likely diagnosis? A. Alzheimer disease B. Huntington disease C. Lewy bodies D. Prion protein E. Vascular disease

A. Alzheimer disease

A 75-year-old was found unresponsive in her house by her neighbor who had come over to help clean her house. An empty unlabeled pill container was found next to her on the bathroom floor. She was rushed to the ER, stabilized, and is now in ICU on a mechanical ventilator. Which of the following are true regarding suicide in the elderly? A. Approximately 75% of the elderly who commit suicide had visited a primary care physician within the preceding month, but their symptoms went unrecognized. B. Elderly persons attempting suicide are more likely to be married and living with their spouse. C. Elderly persons attempting suicide usually report good sleeping habits. D. Firearms are the most common means of suicide in the elderly. E. Suicidal behaviors increase with age, but rates of completed suicides don't.

A. Approximately 75% of the elderly who commit suicide had visited a primary care physician within the preceding month, but their symptoms went unrecognized. Suicidal behaviors DO NOT increase with age, but rates of completed suicides DO. Drug overdose is the most common means of suicide in the elderly.

A 40-year-old male presents to the clinic with a dry cough and wheezing for the past two days. He states that his symptoms began two days ago with a headache for which he took aspirin. He reports no fever but does report some continued shortness of breath. He has smoked 1-2 cigarettes a day for the past six months. Physical exam is negative except for bilateral wheezing and erythema on the face. What is the most likely diagnosis? A. Asthma B. COPD C. Foreign body aspiration D. Pneumonia E. Pulmonary embolism

A. Asthma 21% of adults who have asthma have aspirin-induced asthma and should avoid aspirin and NSAIDS. Even though the patient is a smoker, he is less likely to have a COPD exacerbation because he has only smoked for six months. The patient's duration of symptoms (two days) and reporting no fever lead away from the diagnosis of pneumonia. Also, clinical history gives you very little reason to suspect foreign body aspiration or pulmonary embolism. The diagnosis of asthma could be confirmed with pulmonary function testing.

Which of the following choices correctly identifies where an infant's weight should be at two weeks of age? Choose the single best answer. A. At birth weight B. 2% to 3% below birth weight C. 5% above birth weight D. 10% above birth weight E. Depends entirely on whether the baby is bottle-fed or breast-fed SUBMIT

A. At birth weight Term newborns typically lose weight in the first several days after birth. Weight loss of up to 10% of the birth weight is considered normal. Most newborns regain their birth weight in the first 10 to 14 days, whether fed by breast or bottle. Once gaining weight, neonates typically gain approximately 30 gm (1 oz) per day until three months of age.

A 34-year-old overweight female in good health comes to the clinic for a routine physical. Which of the following screenings are recommended by the U.S. Preventive Services Task Force (USPSTF)? A. Blood pressure screening B. Cholesterol panel C. Complete blood count D. HbA1C E. Thyroid stimulating hormone levels

A. Blood pressure screening

This 45-year-old has never been to a primary care provider. She presents today to establish care and get her health in order. Her concerns today are fatigue, weakness, numbness, insomnia, feeling sad at times, anhedonia, increased appetite, weight gain, dry skin, and increasing hair loss within the past month. Her vital signs are: Pulse is 78 beats/minute Respiratory rate is 18 breaths/minute Oxygen saturation is 95% Blood pressure is 152/84 mmHg Weight is 147 kg (325 lbs) Body Mass Index is 41 kg/m2 Today, her physical exam is significant for thinning hair, poor dentition, a systolic murmur heard at the left upper sternal border, an obese abdomen, and bilateral knee stiffness and pain on range of motion exam. Remainder of the physical exam is within normal limits. Which laboratory tests or studies can be done to rule out medical causes of insomnia, fatigue, and depression? A. CBC, CMP, and TSH B. Chest-X Ray C. CT head without contrast D. HgbA1c, lipid panel, urine microalbumin E. MRI brain with contrast

A. CBC, CMP, and TSH

What is the most likely diagnosis for Amelia at this point? Select the single best choice. A. Colic B. Failure to thrive C. Gastroesophageal reflux D. Infection E. Intussusception F. Milk allergy G. Pyloric stenosis

A. Colic Amelia most likely is experiencing colic (A), as she is gaining weight normally, has normal vital signs, and a normal physical exam. Infants with pyloric stenosis experience projectile vomiting and may have a palpable pyloric mass on abdominal exam. Intussusception (E) is characterized by sudden recurrent periods of severe pain leading to a progressively ill-appearing child. Cow's milk allergy (F) can take a variety of forms, including anaphylactoid reactions, eczema, and proctitis leading to bloody stools. There is some evidence that colic in bottle-fed infants can improve with a change to soy-based or hydrolyzed cow's milk formula, but milk protein allergy is not felt to be the predominant etiology for colic. Gastrointestinal reflux (C) typically leads to regurgitation of milk, but not with the increase in fussiness seen in Amelia's case. Both infection (D) and failure to thrive (B) would present with an ill-appearing child who is not gaining weight as expected.

A 55-year-old male with no significant past medical history and generally healthy behaviors presents to the clinic for a health care maintenance exam. He says, "I'd like to get tested for all types of cancer." He does not have any family history of cancer. Review of systems is negative for any symptoms of prostate cancer, such as urinary frequency, urgency, retention, hematuria, weight loss, or back pain. He is a lifelong non-smoker, and he doesn't drink alcohol or use recreational drugs. Which of the following screening tests is given either a USPSTF A or B recommendation in favor of its routine use for patients such as this one? A. Colon cancer screening B. ECG screening for coronary artery disease C. Lung cancer screening D. Pancreatic cancer screening E. Prostate specific antigen (PSA) testing

A. Colon cancer screening

Ms. Kovacs is a first-time mother you are seeing on postpartum day two in the hospital, with her newborn son Christopher. Her pregnancy was uncomplicated and Christopher was born full-term via spontaneous vaginal delivery with no complications and a birth weight of 7 lbs, 0 oz (3,178 grams). Ms. Kovacs is getting ready to be discharged home today but is concerned because the baby now weighs 6.8 lbs (3,087 grams) and she hasn't been able to produce milk every time Christopher wants to feed. Upon receiving your recommendations, she has been attempting to breastfeed with good technique every two to three hours with strong latching and vigorous suckling. She is still concerned her baby isn't getting enough to eat. Which of the following would be the most appropriate advice for mom? A. Continue to attempt breastfeeding B. Request a lactation consultation to ensure the baby gains weight C. Remain in the hospital until Christopher is having full and consistent feeds with breast milk D. Stop attempting to breastfeed and begin formula E. Switch from breast milk to formula for a few days to allow mom's body to produce more milk, and then attempt breastfeeding again

A. Continue to attempt breastfeeding Given the history, the correct approach is for the mother to continue breastfeeding (A) as it can take up to 72 hours after delivery for significant milk production to begin. It would be inappropriate to supplement with or switch to formula (D). There is no rationale for continued hospitalization as the baby is suckling well on a regular basis that will stimulate appropriate milk production in the mother (B). A lactation consultation is not essential as the baby's weight loss is normal (down only 3% since birth) (C) and as the mother has appropriate technique and the baby is latching and suckling well. However, a visit from a lactation consultant prior to discharge can be a very helpful resource for new moms and can provide additional support should parents encounter breastfeeding problems once they are at home.

Ms. Jones is a first time mother who brings in her 5-day-old son Tommy for his first scheduled newborn visit. Upon reviewing Tommy's hospital records you note that mom's pregnancy was uncomplicated and that Tommy was born full term via spontaneous vaginal delivery and his birth weight was average for gestational age at 3000 grams. Per your discussion with mom, she has been breastfeeding successfully with good technique every two to three hours on demand since delivery. Tommy's physical exam is normal and largely unchanged from the hospital; he is at the 70th percentile for height and head circumference, but his weight has decreased to 2,750 grams. Mom is very concerned by this weight loss and wants to know what she should do. You should recommend: A. Continuing breast feeding with same frequency and return to clinic in 5-7 days B. Hospitalizing Tommy until he shows adequate weight gain for his age C. Increasing the frequency of breastfeeding until Tommy starts gaining weight D. Refer Ms. Jones and Tommy to a lactation consultant E. Supplementing breast milk with 1-2 oz of baby formula until Tommy starts gaining weight

A. Continuing breast feeding with same frequency and return to clinic in 5-7 days It is normal for newborns to lose up to 10% of their birth weight in the first week. Their weight should be back to their birth weight by two weeks of age. Tommy has only lost 8.3% of his birth weight, and breastfeeding has been well established. There is no need to increase the frequency of breastfeeding or to supplement with formula if breastfeeding is successfully established, as it is here. Referral to a lactation consultant is frequently a helpful intervention for parents who are encountering challenges with breastfeeding, but this would not be indicated here.

A 51-year-old female comes to you with acute pain and swelling of the knee. Joint fluid analysis confirms the diagnosis of acute gout. She has a past medical history of atrial fibrillation, hypothyroidism, hypertension, and prior treatment for H. pylori infection. Her current medications include losartan, warfarin, levothyroxine, and omeprazole. She is allergic to penicillin medications. Recent laboratory studies revealed normal hemoglobin and hematocrit, blood urea nitrogen, and creatinine levels. Which of the following information from her history would dissuade you from initiating NSAID therapy? A. Currently on warfarin B. Her age C. Hypothyroidism D. Penicillin allergy E. Previous H. pylori infection

A. Currently on warfarin

"Which lifestyle modification will decrease blood pressure the most?" Choose the single best answer. A. DASH eating plan B. Dietary potassium C. Dietary sodium reduction D. Moderation of alcohol consumption E. Physical activity F. Weight loss of 10 kg or less

A. DASH eating plan

A 35-year-old female with a history of tension-type headaches presents to your office with a headache that is not responsive to 400 mg Ibuprofen TID or 500 mg Tylenol TID. Upon gathering further history you find that in the past her headaches were controlled with these medications but recently have been worse with this same treatment. She complains of 4-5 headaches weekly that are often present on awakening. Which of the following is the best treatment for her disorder? A. Discontinue use of Ibuprofen and Tylenol B. Prescribe a different NSAID to take for acute headaches C. Prescribe an opioid medication to take for acute headaches D. Prescribe propranolol for headache prevention E. Prescribe sumatriptan to take as needed

A. Discontinue use of Ibuprofen and Tylenol The patient in this case is likely experiencing rebound headaches, due to overuse of Ibuprofen and Tylenol. The appropriate treatment for this type of headache is discontinuation of the agents (A). Counseling must be provided that headaches may worsen before resolving over time.

Mr. Gill is a 27-year-old male who presents to his primary care physician with post-prandial epigastric burning. The burning is episodic, without associated hematemesis, dysphagia, or odynophagia. In the past, he has episodically used an OTC chewable calcium carbonate to provide relief. Over the past month, he has had to increase the frequency of its use to four times daily. His blood pressure is 120/71 mm Hg. His pulse is 75 beats/minute and regular. Physical exam reveals minimal epigastric tenderness on palpation but is otherwise normal. Which of the following is the most appropriate next step in management? Choose the single best answer. A. Eight-week trial of pantoprazole once a day B. H. pylori IgG serologic testing C. Fecal immunochemical testing (FIT) D. Fecal occult blood testing (FOBT) E. Upper endoscopy with biopsy

A. Eight-week trial of pantoprazole once a day This patient displays no red flag symptoms at this time. The most widely accepted initial intervention in a patient like this is empiric treatment with a histamine-2 receptor blocker or a proton pump inhibitor such as pantoprazole.

A 65-year-old female presents to your office for a routine visit. She is found to have a blood pressure of 146/96 mmHg. You repeat the blood pressure in her other arm and get 148/92 mmHg. Her pulse is 70 and regular. Her last BP reading was one year ago and was 120/76 mmHg. She has no other medical problems. Her BMI is 28. She states that she likes to walk 30 minutes every other day with her husband and has been doing that for years now. What is the most appropriate diagnosis at this time? A. Elevated blood pressure reading B. Secondary hypertension C. Stage 1 hypertension D. Stage 2 hypertension E. White coat hypertension

A. Elevated blood pressure reading

A 59-year-old patient comes to the local ER with a swollen, tender knee that started yesterday. He returned home two days ago from a five-day hike on the Appalachian Trail. He reports no recent or previous injury to the knee and any history of previous inflammatory joint disease. Vital signs: temperature is 36.5 C (97.7 F), pulse is 80 beats/minute, blood pressure is 139/75 mmHg, and respiratory rate is 22 breaths/minute. His lower extremities are marked with abrasions in various stages of healing. He holds the knee in full extension. The knee is swollen, reddened, and tender, and it feels warm. Which of the following is the most appropriate next step? A. Knee aspiration B. Magnetic resonance imaging (MRI) of the knee C. Prescription for colchicine D. Prescription for full-dose acetaminophen E. X-ray of the knee, including sunrise and standing films

A. Knee aspiration

You are seeing a 19-year-old female who suffered a knee injury playing soccer one day ago. The injury involved a sudden deceleration in which she planted her right foot in front of her while running, whereupon another athlete fell against her shin. She felt a pop and sudden pain. She had to be helped off the field and her knee swelled immediately. Today, she reports that she has considerable right knee pain with bearing weight and that her knee feels unstable at times. Her past medical history is unremarkable, and she takes no medications. On exam, her vital signs are perfectly normal. You conduct a knee exam. Which of the following exam maneuvers is most likely to be abnormal in this patient? A. Lachman test B. Laxity to valgus stress C. Laxity to varus stress D. McMurray test E. Posterior drawer test

A. Lachman test

A 52-year-old female with no past medical history presents to your office with amenorrhea. The patient states that her menstrual cycles previously occurred approximately every 28 to 34 days. However, she has not had a menstrual cycle for the last 10 months. She also reports insomnia and intermittent dysuria. She reports no headaches, abdominal pain, constipation or diarrhea, changes in hair distribution, or easy bruising. She has lost 15 pounds since her last visit eight months ago, which she attributes to improving her diet and beginning regular exercise. Which of the following tests or pairs of tests is used to confirm menopause? A. Luteinizing hormone (LH) and follicle-stimulating hormone (FSH) B. Prolactin and Thyroid Stimulating Hormone (TSH) C. Prothrombin time (PT) and international normalized ratio (INR) D. Testosterone and dehydroepiandrosterone sulfate (DHEA-S) E. Thyroid-stimulating hormone (TSH) and free T4

A. Luteinizing hormone (LH) and follicle-stimulating hormone (FSH)

A 26-year-old professional football player comes to the clinic with the concern of hair loss. On examination, the scalp is scaly and erythematous, and certain regions are purulent. There are several circular spots where the hair follicles are no longer present. KOH of skin shows hyphae. What is the most appropriate next step in the management of this patient? A. Oral griseofulvin B. Oral prednisone C. Punch biopsy of lesion D. Topical griseofulvin E. Topical prednisone

A. Oral griseofulvin

You are seeing a 55-year-old male who presents to the family medicine practice with a two-week history of daily episodes of sudden lightheadedness and palpitations. These seem to occur without provocation and last for approximately five minutes each time. He has not passed out during these episodes, but he has felt the need to sit down when they occur. Sitting calmly and waiting seems to make them go away. He reports no chest pain, diaphoresis, jaw pain, or arm pain with these episodes. He has a past medical history of gout and hypertension, for which he takes daily allopurinol and losartan. He does not smoke or drink. He is not under any recent stresses at home or work. On exam, he is in no acute distress. His vital signs are completely normal, as is his cardiopulmonary exam. A recent complete blood count and TSH were normal. Which of the following is the most appropriate next step? A. Order a 48-hour Holter monitor B. Order an echocardiogram C. Order an exercise stress test D. Reassure him that his palpitations are likely benign and that he should focus on a healthy lifestyle E. Transfer him to the emergency department for admission to the hospital

A. Order a 48-hour Holter monitor This patient is experiencing frequent palpitations that are symptomatic enough to make him lightheaded and need to sit down. Given his description of his symptoms and his normal exam, episodic arrhythmia is the most likely cause of his symptoms. As such, a 48-hour Holter monitor (A) would be the most appropriate test. He is having daily symptoms, so a longer-term loop monitor is not needed in this situation. Echocardiography (B) and cardiac stress testing (C) would be appropriate tests to evaluate for coronary artery disease or congestive heart failure. These are less likely explanations of his current symptoms than an arrhythmia. Reassuring him that these episodes are benign could risk missing an important diagnosis (D). The patient is stable and is not having symptoms or findings at the time of the visit, so referring him to the ED (E) is not appropriate.

A 38-year-old female with a past medical history of sarcoidosis and recent completion of a six-month steroid taper presents to her primary care physician after two weeks of lower lumbar back pain. She does not recall any trauma but began to feel a sharp pain after bending over to pick up laundry. The pain radiates bilaterally into her anterior abdomen. She has found no relief with over-the-counter NSAIDs. On physical exam, she has point tenderness along her vertebrae in the L1-L2 region. There are no neurologic deficits and reflexes are intact. Which of the following is the most appropriate next step in management? A. Order a plain x-ray B. Order complete blood count (CBC) C. Reassess in four weeks D. Recommend conservative management E. Refer to spine specialist

A. Order a plain x-ray

Mr. and Mrs. Thomas are first-time parents who have brought their 5-day-old newborn son into your office for his first visit. Mom's pregnancy, baby birth, and today's physical exam are all normal. Both parents are sharing the duties of feeding, cleaning, holding, and watching their son without any problems. When Mrs. Thomas steps out of the room to use the restroom, Mr. Thomas asks to speak with you privately. He wants to know if you can prescribe some antidepressant medication for his wife because since giving birth she hasn't been the same. She has been having mood swings multiple times a day, he has found her crying in bed at night after putting their son in his crib, and she gets very irritated and yells at her parents any time they try to help with the baby. What would be the most appropriate advice to give Mr. Thomas? A. This is known as the "postpartum blues"; is quite common for new moms; and is very likely to self-resolve. B. This behavior is very common and a mild antidepressant can help most women get through it. C. The behavior is uncommon at five days postpartum and mental health counseling should be considered. D. This behavior sounds dangerous and hospitalization is in order. E. You should challenge Mr. Thomas on whether he is helping enough with child care.

A. This is known as the "postpartum blues"; is quite common for new moms; and is very likely to self-resolve.

Which of the following are milestones of normal development that you might expect to see in a four-week-old infant? Select all that apply. A. Can flip from back to front while supine. B. Can pick up head and move from one side to other when lying prone. C. Diminishment of jerky upper extremity motions. D. Hearing maturation; beginning to turn head toward familiar sounds. E. Increased ability to focus on faces and show preference for human faces. F. Increased alertness and responsiveness. G. Move hands to mouth and eye range.

B, C, D, E, F, G. Flipping from back to front is generally not achieved until at least four months of age (A).

While the patient is standing for a back exam, what should be examined? Select all that apply. A. Ask the patient to jump B. Ask the patient to squat C. Ask the patient to walk D. Check flexion of the spine E. Inspect the curvature of the spine F. Palpate the paraspinal muscle

B, C, D, E, F.

Given that PUD is our primary concern at this point, but we are still considering functional dyspepsia, which of the following are appropriate next steps in diagnostic and therapeutic planning? Select all that apply. A. Obtain a complete blood count (CBC) B. Order an H. pylori fecal antigen test C. Order H. pylori IgG serology D. Refer for a urease breath test E. Send him home with a fecal occult blood test (FOBT) F. Start tricyclic antidepressants to modulate visceral sensation and minimize pain

B, C, D, E.

Which of the following are risk factors for osteoporosis? Select all that apply. A. BMI > 30 B. Early menopause C. History of previous fracture as an adult D. Sedentary lifestyle E. Smoking

B, C, D, E. Higher BMI (A) seems to be protective by increasing estrogen levels.

Mr. Payne asks Dr. Lee: "What's the likelihood that this back pain will go away completely?" Select all that apply. A. It is more common for patients with psychosocial distress to recover. B. Longer time to recovery is associated with older patients. C. Most back pain is improved in 4 to 6 weeks. D. Recurrence rate for back pain varies from 35% to 75%.

B, C, D.

Which factors, in addition to finances, may have contributed to Mr. Rodriguez not seeking medical care in the recent months? Select all that apply. A. He has so many other issues to worry about he doesn't have time to worry about his overall health. B. He may be an undocumented immigrant, fearing deportation or discrimination. C. He may view the U.S. health care system as unfriendly and intimidating. D. He may view Western medicine as non-holistic. SUBMIT

B, C, D.

Which of the following contraceptive methods are possible options now for a patient who just gave birth? Select all that apply. A. Combination oral contraceptive B. Copper-containing intrauterine device C. Injectable medroxyprogesterone acetate (Depo-Provera) D. Progestin-only oral contraceptive SUBMIT

B, C, D. According to CDC guidelines, combination oral contraceptives (A) are considered inappropriate in the first 21 days after delivery because of the increased risk of deep vein thrombosis

Which of the following are symptoms of acute sinusitis? Select all that apply. A. Clear nasal drainage B. Facial pain C. Fever D. Gradual worsening five days after the onset of a viral upper respiratory infection E. Headaches F. Initial improvement after a viral upper respiratory infection followed by a worsening of symptoms G. Nasal congestion or obstruction for more than 12 weeks H. No improvement with decongestants I. Toothache SUBMIT

B, C, E, F, H, I. Clear nasal discharge (A) is not typical of acute sinusitis—the drainage is usually opaque and mucopurulent. Gradual worsening in the first five days after the onset of a URI (D) is fairly typical for the common cold and other viral infections, not sinusitis. Nasal congestion lasting more than 12 weeks (G) is consistent with chronic sinusitis, not acute.

Mr. Dennison is studying the "My Asthma Plan" and he asks you to talk more about how he would feel when his peak flow reading is in the Red Zone. Which of the following are examples of symptoms and signs indicating he is in the Red Zone? Select all that apply. A. Can do some activities, but not all B. Gray or blue lips or fingernails C. Peak flow reading that is < 50% of his personal best reading D. Quick-relief medication such as a short-acting beta2 agonist inhaler helps temporarily E. Very short of breath, including when walking or talking

B, C, E. As described in "My Asthma Plan", a patient in the red zone would be very short of breath, including when walking or talking (E) and signs of cyanosis (B) could be present and indicate severe respiratory distress. In addition, the peak flow reading would be < 50% of the personal best (C). For some patients, the peak flow reading is a more reliable indicator of severity than symptoms are. A peak flow-based plan may be most useful for patients who have difficulty perceiving airflow obstruction, or who have a history of severe exacerbations. A patient in the red zone would also be unable to perform usual activities and would not improve with quick-acting medications such as short-acting beta2-agonists. Therefore, answers A and D are not correct.

Dr. Hill asks, "What do you think are the three most likely causes of Savannah's 1st trimester vaginal bleeding?" Select all that apply. A. Cervical abnormalities (e.g., excessive friability, malignancy, polyps, trauma) B. Ectopic pregnancy C. Idiopathic bleeding in a viable pregnancy D. Molar pregnancy (gestational trophoblastic disease) E. Spontaneous abortion (miscarriage) F. Vaginal trauma

B, C, E. Typically, molar pregnancies have a characteristic appearance on ultrasound and are associated with markedly increased (>100,000 mIU/mL) quantitative hCG levels.

In the hallway, Dr. Wilson asks, "Since Ms. Sanchez has not been seen for her diabetes for over a year, which of the following tests are appropriate to order?" Select all that apply. A. 24-hour urine protein and glucose B. Fasting lipid profile (total cholesterol, LDL- and HDL-cholesterol and triglycerides) C. Fingerstick blood glucose D. Hemoglobin A1C E. Liver enzyme tests (LFTs) F. Serum B12 levels G. Serum creatinine and calculated GFR H. Spot urine albumin/creatinine ratio I. Thyroid-stimulating hormone (TSH)

B, D, E, F, G, H. Ms. Sanchez would be due for a lipid profile (B), hemoglobin A1C (D), Serum B12 level (F) since she is on metformin, serum creatinine and GFR (G), spot urine albumin/creatinine ratio (H), and LFTs (E). Lipid profiles have historically been done fasting, though there is growing evidence that they may be done without this restriction.

Which of the following agents have been proven to cause or contribute to the development of peptic ulcer disease? Select all that apply. A. Acetaminophen B. Aspirin C. Caffeine D. Cigarette smoking E. Helicobacter pylori F. Ibuprofen G. Moderate physiologic stress

B, D, E, F, G.

Which of the following statements are true about thiazide diuretics? Select all that apply. A. A 50 mg dose of hydrochlorothiazide reduces blood pressure and decreases morbidity and mortality more than a 25 mg dose of hydrochlorothiazide. B. Thiazide diuretics may cause increases in blood sugar levels. C. Thiazide diuretics should be started at 25 mg in older adult patients. D. Thiazide diuretics should be used with caution in patients with a history of gout. E. Thiazides may affect electrolyte levels. F. Thiazides may be taken in the evening.

B, D, E, F. A) Studies have also demonstrated that despite the availability of 50 mg hydrochlorothiazide tablets, doses of hydrochlorothiazide above 25 mg do not decrease blood pressure further or further reduce morbidity and mortality rates. C) Thiazide diuretics should be started at lower doses such as 6.25 mg or 12.5 mg/d in older adult patients because this population may be more sensitive to this drug class and may experience hypotensive episodes or electrolyte abnormalities. Most other adults can start at 25 mg/d.

Which tests would you order to rule out other causes for symptoms of insomnia, fatigue, and a depressed mood? The best options are indicated below. Your selections are indicated by the shaded boxes. A. Brain CT scan B. CBC C. Chest x-ray D. Complete metabolic panel E. TSH F. Urinalysis

B, D, E.

According to current immunization guidelines, what immunizations should you offer to Mr. Dennison? (40 YO with asthma) Select all that apply. A. Haemophilus influenzae type b conjugate vaccine (Hib) B. Influenza vaccine C. Pneumococcal conjugate (valent-13) vaccine (PCV13) D. Pneumococcal polysaccharide (valent-23) vaccine (PPSV) E. Tetanus, diphtheria, acellular pertussis vaccine (Tdap)

B, D, E. According to the CDC Immunization guidelines, Mr. Dennison should receive the influenza vaccine (B) and tetanus, diphtheria, acellular pertussis vaccine (E) if due. Due to his asthma (chronic lung disease), he should also be offered the pneumococcal polysaccharide (PPSV) vaccine (D). The Haemophilus influenzae type B conjugate vaccine (A) is recommended for all children and for adults with certain medical conditions (but not lung disease). The pneumococcal polyconjugate valent (PCV13) vaccine (C) is indicated for children and adults with certain medical conditions (but not lung disease). Therefore, answers (A) and (C) are incorrect.

Which of the following medical conditions is associated with depression? Select all that apply. A. Asthma B. Dementia C. Hypertension D. Hypothyroidism E. Parkinson disease

B, D, E. Hypertension (C) and asthma (A) have not been specifically linked to higher rates of depression.

Looking at 55 YO Mrs. Parker's medical record, you see that her last Papanicolaou (Pap) with cotest for HPV was four years ago and mammogram was two years ago. They were both normal. Her last FIT (Fecal Immunochemical test) was 18 months ago and normal as well. Which of the following are recommended screening exams for Mrs. Parker at this time? Select all that apply. A. CA-125 B. Colon cancer screening C. DEXA scan D. Mammogram E. Pap test

B, D.

Of the following, which are considered instrumental activities of daily living (IADL)? Select all that apply. A. Bathing B. Doing laundry C. Dressing D. Maintenance of continence E. Managing finances F. Managing medications G. Preparing meals H. Transferring from bed to chair (or vice versa) I. Using a telephone

B, E, F, G, I.

Dr. Medel asks you, "Which of the following accurately describes the best way to measure blood pressure?" Select all that apply. A. An adult-sized cuff should be used for most adults. B. The length of the bladder of the cuff must be at least 80% of the arm circumference. C. The patient should be seated quietly for 30 minutes before a blood pressure measurement is taken. D. The patient should be seated on an examination table when a blood pressure is taken. E. The patient's arm should be supported at heart level.

B, E.

Which of the following are approved initial treatment regimens for the eradication of H. pylori? Select all that apply. A. PPI therapy for an additional 4 weeks for 8 weeks of total therapy B. Standard dose PPI + amoxicillin 1 gram + clarithromycin 500 mg all twice daily for 10 to 14 days C. Standard dose PPI + amoxicillin 1 gram + levofloxacin 500 mg all twice daily for 10 days D. Standard dose PPI + trimethoprim/sulfamethoxazole double strength + erythromycin 500 mg all twice daily for 10 to 14 days E. Standard dose PPI once or twice daily + metronidazole 250 mg, tetracycline 500 mg, + bismuth subsalicylate 525 mg each four times daily for 10 to 14 days

B, E.

After you leave the room, Dr. Medel takes a moment to ask you a question. "If Ms. Rios had had an elevated fasting or one-hour glucose measurement, we would want to follow up with a three-hour glucose tolerance test (3hr GTT). This test includes a fasting glucose, a 100 g glucose load, then a glucose at one, two, and three hours post-prandial. What determines when a 3 hr GTT is abnormal, allowing you to diagnose Gestational Diabetes?" Choose the single best answer. A. One of the four glucose measurements exceed the cutoffs. B. Two or more of the four glucose measurements exceed the cutoffs. C. Three or more of the four glucose measurements exceed the cutoffs. D. All four of the four glucose measurements exceed the cutoffs.

B.

How long do postpartum blues typically last? Choose the single best answer. A. One week B. Two weeks C. Four weeks D. Eight weeks

B. Postpartum blues typically last about two weeks after delivery. Longer lasting or more severe symptoms raise a concern of postpartum depression.

Should you refer Mr. Rodriguez to a gastroenterologist at this point without alarm signs or symptoms suggesting GI bleeding or cancer? A. Yes B. No

B. Since there are no alarm signs or symptoms suggesting GI bleeding or cancer, a referral to a gastroenterologist is not warranted. Further workup can continue in the office setting.

A 32-year-old female presents at your office for a preconception health visit. She is a G2P2, both deliveries were vaginal. Her first child was born with a neural tube defect. According to the USPSTF, what dosage of folate should this patient take daily before she gets pregnant? A. 1 mg B. 4 mg C. 8 mg D. 400 mcg E. 800 mcg

B. 4 mg The U.S. Preventive Services Task Force (USPSTF) recommends that all women planning or capable of becoming pregnant take a 400 to 800 mcg daily supplement of folate to prevent neural tube defects. The recommendation increases to 1 mg daily in patients with diabetes and epilepsy. In patients who have previously had a child with a neural tube defect, the recommendation increases to 4 mg daily.

You are seeing a 34-year-old cisgender female with a history of eczema, seasonal allergies, and asthma. She is presenting for a follow-up of her asthma, which has been bothersome in the past four months. She has been needing her albuterol rescue inhaler several times per week, and she had a recent visit to the emergency department for an asthma exacerbation. She currently takes a moderate-potency inhaled corticosteroid twice daily and uses an albuterol metered-dose inhaler as needed. She demonstrates proper technique for self-administering these medications with a spacer. She also takes loratadine and intranasal fluticasone for her allergic rhinitis. She does not smoke, drink alcohol, or use illicit substances. She works as a bank teller and lives with her partner, who also does not smoke. They have no pets. On exam, her vitals are normal, her head and neck exams are normal, and she has very mild expiratory wheezing diffusely. Of the following, which is the most appropriate next step? A. Add a leukotriene receptor antagonist B. Add a long-acting beta agonist to her twice-daily inhaled corticosteroid C. Add theophylline D. Change the delivery of her inhaled medications from metered-dose inhalers to nebulized delivery. E. Refer her to an allergist for allergy testing and possible immunotherapy

B. Add a long-acting beta agonist to her twice-daily inhaled corticosteroid

A 61-year-old G4P4 female presents to a local emergency room with vaginal bleeding. The bleeding began two days ago and is described as spotting. She began her menses at age 16 and had regular menstrual cycles until the age of 59. She reports smoking ½ pack per day for the last 23 years and drinks 1-2 glasses of wine with dinner every evening. She reports no abdominal pain or dysuria. On exam, she has a blood pressure of 124/76 mmHg and a BMI of 18.2 kg/m2. Her pelvic exam reveals a small amount of dried blood in the vaginal vault and mildly atrophic vaginal mucosa. Which one of the following characteristics of this patient increases her risk of endometrial cancer? A. Age of menarche B. Age of menopause C. Body habitus D. Multiparity E. Smoking

B. Age of menopause

A 33-year-old G0P0 female presents to your clinic for her first prenatal visit. Her home pregnancy test was positive, she has been experiencing mild nausea for two weeks. Her last normal menstrual period was six weeks ago. What test should her initial prenatal visit include? A. Abdominal ultrasound B. Complete blood count C. Complete metabolic panel D. Glucose challenge test E. Quad screen

B. Complete blood count An initial pregnancy evaluation should include CBC, RPR, HIV, Rubella, Blood type, and Hepatitis B. A CBC is included to test for nutritional and congenital anemias as well as platelet disorders. The quad screen typically occurs later in the pregnancy (15-20 weeks) and transvaginal ultrasound (not abdominal ultrasound) is more reliable for dating pregnancy at six weeks gestation, though it is not needed routinely. A complete metabolic profile is not a routine part of an initial pregnancy evaluation and should be ordered only when indicated. A glucose challenge test, in an average-risk prenatal patient like this one, is a screening test performed at 24 to 28 weeks gestational age, and not at the first prenatal visit.

A 54-year-old male with a history of chronic gout and GERD presents to your office for his health maintenance exam. Vital signs today are blood pressure 138/88 mmHg, pulse 65 beats/min, respiratory rate 12 breaths/minute, BMI 29 kg/m2. He does not smoke cigarettes or use illicit substances, and he does drink one or two glasses of wine most evenings. He currently jogs three times a week for approximately 30 minutes at a time. He and his partner order takeout food for supper twice per week and otherwise cook at home. He does not particularly like vegetables, but he tries to eat a piece of fruit every day. He has no current concerns, review of systems is negative, and his physical exam is unremarkable. You recommend lifestyle changes. Which of the following changes is most likely to improve his cardiovascular risk? A. Alcohol cessation B. DASH eating plan C. Increased exercise D. Increasing dietary potassium E. Supplementation with vitamin D

B. DASH eating plan

A 32-year-old patient at 34 weeks and 5 days gestation (G2P1) presents to the clinic with headache and RUQ abdominal pain. Blood pressure is 172/121 mmHg on examination while seated. No visual changes noted. Edema is present in the hands, bilaterally. Urine dipstick demonstrated 4+ protein. FHT are 117. Which of the following is the most appropriate next step in the management of this patient? A. Daily aspirin B. Expedited delivery of the premature fetus C. Lisinopril D. Strict bed rest until 37 weeks E. Twice-weekly non-stress testing SUBMIT

B. Expedited delivery of the premature fetus

A 32-year-old patient at 34 weeks and 5 days gestation (G2P1) presents to the clinic with headache and RUQ abdominal pain. Blood pressure is 172/121 mmHg on examination while seated. No visual changes noted. Edema is present in the hands, bilaterally. Urine dipstick demonstrated 4+ protein. FHT are 117. Which of the following is the most appropriate next step in the management of this patient? A. Daily aspirin B. Expedited delivery of the premature fetus C. Lisinopril D. Strict bed rest until 37 weeks E. Twice-weekly non-stress testing

B. Expedited delivery of the premature fetus Expedited delivery of the fetus is the best treatment for preeclampsia with severe features. (Note: preeclampsia with severe features is not necessarily an indication for a C-section, however.)

A 60-year-old female with a past medical history of chronic gout, depression, and stage 1 hypertension presents to your office for a follow-up visit. She has been attempting to reduce her blood pressure with behavioral changes but has had difficulty maintaining the changes. Today, her vital signs are blood pressure 144/90 mmHg, pulse 78 beats/minute, respiratory rate 12 breaths/minute. Recent basic metabolic panel was completely normal. As you consider starting a medication for her hypertension, which of the following medications is most likely to cause an adverse event in this patient? A. Amlodipine B. Hydrochlorothiazide C. Lisinopril D. Losartan E. Spironolactone

B. Hydrochlorothiazide

A 21-year-old G1P0 patient presents to the clinic as a new patient to establish prenatal care. Which statement represents something that would not be expected to be a benefit of group prenatal care for this patient? A. Decreases the likelihood of preterm delivery B. Increases adherence to techniques for pain management during labor C. Increases physician contact D. Increases support network E. Shared education between patients

B. Increases adherence to techniques for pain management during labor

A 21-year-old G1P0 patient presents to the clinic as a new patient to establish prenatal care. Which statement represents something that would not be expected to be a benefit of group prenatal care for this patient? A. Decreases the likelihood of preterm delivery B. Increases adherence to techniques for pain management during labor C. Increases physician contact D. Increases support network E. Shared education between patients

B. Increases adherence to techniques for pain management during labor

A 24-year-old G1P0 patient at 14 weeks presents with vaginal bleeding and abdominal cramping. On examination her vital signs are: blood pressure 120/75 mmHg, pulse 74/minute, temperature 36.9 C (98.4 F) taken orally, respiratory rate 18/minute, and oxygen saturation 99% on room air. On pelvic examination, there is a small amount of blood in the vagina, the cervical os is open, and there is no cervical or adnexal tenderness noted. On pelvic ultrasound, an intrauterine gestational sac with a yolk sac is seen. What is her diagnosis? A. Complete abortion B. Inevitable abortion C. Missed abortion D. Septic abortion E. Threatened abortion

B. Inevitable abortion (cervical os is open or dilated, bleeding in early pregnancy) When the cervical os is open or dilated, bleeding in early pregnancy is classified as an inevitable abortion. When the cervical os is closed with bleeding before 20 weeks, the correct diagnosis is a threatened abortion. A missed abortion is fetal demise without cervical dilation. Complete abortion occurs when products of conception have been completely expelled from the uterus. Septic abortion is highly unlikely in the setting of her being afebrile and pain free.

A 48-year-old male with a past medical history that includes hypertension, chronic obstructive pulmonary disease (COPD), and hyperlipidemia presents to the clinic as a new patient in October for a general physical exam. History reveals that he has been smoking a pack of cigarettes daily since age 20. He drinks two beers daily. He is intermittently nonadherent with his medications. A review of the state immunization database reveals that the only immunization he has received as an adult was a tetanus diphtheria shot administered 12 years ago. Which of the following vaccine combinations would be most appropriate for this patient? A. Influenza, meningococcal, and zoster B. Influenza, pneumococcal, and Tdap C. Influenza, zoster, and Tdap D. Meningococcal, pneumococcal, and Tdap E. Meningococcal, pneumococcal, and zoster

B. Influenza, pneumococcal, and Tdap

You are seeing a 92-year-old male in the hospital. He was admitted after a fall in which he broke his hip. It is now post-op day two from surgery to repair the fracture. His children report that he has been very confused this morning, with varying levels of alertness. Yesterday, he had been recovering well and even participated in physical therapy. Today, he does not know where he is and is at times combative with his care team. His medical history is positive only for hypertension. His medications include amlodipine 5 mg daily and morphine 4 mg IV every 4 hours as needed for pain. His vital signs are all normal today, and his general exam is unremarkable except for expected postoperative changes to his hip. He is somnolent during the examination. Today's labs include a normal CBC and basic metabolic panel. A urinalysis reveals no leukocyte esterase and is nitrite negative. What is the most likely cause of his current mental status? A. Acute stroke B. Morphine C. Pneumonia D. Residual effects of the anesthetic medications from his surgery E. Urinary tract infection

B. Morphine Morphine (B) and other sedating medications (such as other opioids, benzodiazepines, anticholinergics, etc.) commonly cause delirium in older patients, particularly after the cumulative effects of repeated doses. Discontinuing the pain medication may not be an option, due to his need for pain control, though reducing the dose and careful attention to the dosing frequency may help minimize his symptoms. Acute stroke (A), while sometimes the cause of acute mental status change, is unlikely in a patient with no new physical findings to suggest stroke, such as new motor weakness.

A 68-year-old male with a past medical history significant for hypertension and diabetes presents to your office with a three-month history of headaches. He describes a pain that has occasionally awoken him from sleep and is often worse in the morning. He reports no weakness or changes in vision. His exam shows 4+ reflexes in the right upper and lower extremities but is otherwise normal. What is the best next step in management for this patient? A. Initiate treatment with sumatriptan B. Neuroimaging C. Reassurance D. Referral to counselor for relaxation exercises E. Sleep study

B. Neuroimaging This patient exhibits several red flags that are of concern for intracranial pathology: age over 50, headaches awakening him from sleep, and abnormal reflexes. Because of these concerning features, it is important to perform neuroimaging (B) without delay. Thus, reassurance (C) is inappropriate.

A 27-year-old G0P0 female presents to your office with abdominal pain, fever and vaginal discharge. The discharge has an abnormal odor. She reports a new, male, sexual partner. On pelvic exam, there is cervical motion tenderness and adnexal pain during the bimanual exam. The strings from her IUD, placed two years ago, are seen coming from the os. A cervical culture is positive for Chlamydia trachomatis. What risk factor for chlamydia infection is present in this patient? A. Age B. New sexual partner C. Nulliparity D. IUD E. Sexual orientation

B. New sexual partner

A 63-year old cisgender female comes into your office for her annual preventive exam. She has hypertension and type 2 diabetes. She is not sexually active. Her blood pressure is 125/80 and her physical exam otherwise is within normal limits. You recommend influenza and zoster vaccination. Her last colonoscopy was eight years ago and her last mammogram one year ago; both were normal. She has never had an abnormal Pap test. At the age of 45 she had a total hysterectomy for fibroids. Of the details provided about this patient, which is an appropriate reason to explain why she does not need a Pap test today? A. She experienced menopause more than 10 years ago B. She had a total hysterectomy for fibroids C. She has never had an abnormal Pap test D. She is 63 years old E. She is not sexually active

B. She had a total hysterectomy for fibroids

A 61-year-old patient has recently been diagnosed with type 2 diabetes. Her fasting glucose was 240 mg/dL and her A1C was 8.9%. Her BP has been 148/90 and 146/86 at two separate office visits. Her home BP measurements have been in a similar range. Her creatinine is 0.9 and she has no known heart disease. She currently takes losartan 100 mg daily for a diagnosis of hypertension. Which of the following would be the most appropriate step in managing this patient's blood pressure? A. Make no changes to her medications as her blood pressure is at goal. B. Start amlodipine daily. C. Start furosemide daily. D. Start lisinopril daily.

B. Start amlodipine daily.

Ms. Brady, a 78-year-old female presents to your office after six months for follow-up. Her interval history is significant for a TIA three months ago. Today her Montreal Cognitive Assessment (MoCA) is 19. You note that six months ago her MoCA was 22, and nine months ago it was 26. Physical exam shows temperature of 37.1 C (98.8 F), blood pressure of 167/95 mmHg, pulse of 76 beats/minute, respiratory rate of 14 breaths/minute, and BMI of 23. Chest: regular rate and rhythm, no murmurs; lungs: clear to auscultation; neuro: weakness in the right upper extremity; abdomen: soft, non-tender. She takes atorvastatin and aspirin. Lab studies show Hgb A1c: 5, TSH: 3, B12: 500 pg/mL. Which of the following is the most important recommendation to prevent further disability in this patient? A. Order a mammogram B. Start hydrochlorothiazide C. Start metformin D. Start vegetarian diet E. Weight loss

B. Start hydrochlorothiazide

A 55-year-old female with exercise-induced asthma, but no other chronic health problems, presents to the office with left shoulder pain. She is a tennis player, left-handed, and notices pain serving the ball. When you suggest that it might be her rotator cuff your preceptor asks, "what muscles make up the rotator cuff?" A. Supraspinatus, infraspinatus, teres major, subscapularis B. Supraspinatus, infraspinatus, teres minor, subscapularis C. Supraspinatus, infraspinatus, teres minor, deltoid D. Supraspinatus, infraspinatus, teres major, teres minor E. Supraspinatus, infraspinatus, teres minor, rhomboid minor

B. Supraspinatus, infraspinatus, teres minor, subscapularis

A 60-year-old comes into the clinic for a walk-in appointment. She is tearful and is carrying a box of tissues in her hand. She says she doesn't know why but she has been very sad of late. Reports trouble falling asleep and staying asleep. Used to be the head of her Bridge club, but quit two weeks ago and doesn't feel like going out anymore. She has lost interest in walking her dog, and now just allows him to use the doggie door to let himself out. Feels weak and fatigued and no longer has the energy to do her gardening or shopping. Spends most of her day on the sofa crying while watching TV. She also reports a greatly diminished appetite. Denies suicidal or homicidal ideation, but she does have a history of a previous suicide attempt following her divorce seven years ago for which she was hospitalized. A recent CBC, CMP, CXR, TSH, U/A, and CT of the head were all within normal limits. How long do the above symptoms need to be present in order to make the diagnosis of Major Depressive Disorder? A. One week B. Two weeks C. Four weeks D. Five weeks E. Eight weeks

B. Two weeks

What are common side effects associated with selective serotonin reuptake inhibitors (SSRIs) and serotonin-norepinephrine reuptake inhibitors (SNRIs)? Select all that apply. A. Arthralgias B. Cardiac arrhythmias C. Headaches D. Insomnia E. Nausea

C, D, E. Common side effects of SSRI/SNRIs include: Headaches Sleep disturbances (drowsiness and, less frequently, insomnia) Gastrointestinal problems such as nausea and diarrhea Sexual dysfunction

Based on her migraine history, which two of these would be the best options for Sarah? Select all that apply. A. Acetaminophen/butalbital/caffeine B. Aspirin/butalbital/caffeine C. Ergot alkaloids D. Triptans SUBMIT

C, D.

What are the best medications to offer the asthma patient at this point? Select all that apply. A. Cough suppressant such as codeine B. Cough suppressant such as dextromethorphan C. Inhaled bronchodilator such as albuterol D. Inhaled corticosteroid such as fluticasone E. Nasal corticosteroid such as fluticasone F. Oral antihistamine such as cetirizine

C, E, F. Asthma is high on the differential for Mr. Dennison, but since his diagnosis has not yet been confirmed, it is reasonable to start with an albuterol inhaler and allergy control. If he were unable to have spirometry done in a timely fashion, a trial of an inhaled corticosteroid would also be reasonable at this time.

A 35-year-old recent immigrant who is uninsured returns for follow up. She has a history of SLE treated with NSAID therapy and presented four weeks ago with "heartburn". At that time, she reported episodic, post-prandial epigastric burning without radiation for three months. She reports no nausea, vomiting, melena, weight loss, or fever, and physical exam was significant only for mild epigastric tenderness. Serologic testing for H. pylori IgG was positive, and she began triple therapy, which she completed successfully. Today she reports resolution of symptoms and physical exam is negative for epigastric tenderness. Which of the following is an appropriate test to confirm H. pylori eradication? Choose the single best answer. A. Endoscopy with biopsy B. Repeat serology C. Stool antigen D. Testing for eradication is not indicated

C. Serology (H. pylori IgG) will remain positive in patients with previous infection/exposure. Endoscopy with biopsy is invasive, and not indicated without red flags, such as persistent symptoms despite treatment, melena, or weight loss. Stool antigen and also urea breath tests are equally sensitive for determining eradication. Eradication is indicated because people on chronic daily NSAID therapy are at high risk of ulcer and GI bleed if they continue to have H.pylori.

Which of the following patients is an appropriate candidate for an exercise stress test? A. A 44-year-old female with a BMI of 40 kg/m2 and history of asthma B. A 48-year-old female with a history of intermittent anginal episodes that have been controlled on medications C. A 52-year-old female with new atypical chest pain with a history of elevated cholesterol, smoking and family history of coronary artery disease D. A 58-year-old male who presents to the emergency room with constant substernal chest pressure, diaphoresis and shortness of breath

C. A 52-year-old female with new atypical chest pain with a history of elevated cholesterol, smoking and family history of coronary artery disease

14-year-old Jennifer is brought to the clinic by her mother who is concerned about ADHD. She states Jennifer lacks motivation, regularly does not clean her room and has to be reminded to do chores around the house. She also is concerned Jennifer does not listen to her and avoids doing homework. Which of the following would most strongly support additional evaluation to assess for ADHD? A. A diagnosis of major depression B. A report card with good grades C. A teacher's report of inattentive behavior D. Fidgety behavior in the car when driving long distances E. Symptom onset with entry to high school

C. A teacher's report of inattentive behavior

Ms. Martinez, a 74-year-old female, is brought to your clinic by her husband. He is concerned because his wife, who used to take pride in keeping the house in good shape, has stopped cleaning and caring for her appearance. She recently went shopping and left the stove on. She frequently misplaces her car keys and checkbook. Ms. Martinez reports trouble concentrating and does not have much energy. She has a poor appetite and no longer enjoys knitting. Her Montreal Cognitive Assessment (MoCA) is 28/30. Physical exam: General appearance: well-nourished, appears stated age; HEENT: pupils equal, round and reactive to light, moist mucous membranes; chest: normal S1 and S2, no murmurs; lungs: clear to auscultation bilaterally; abdomen: soft, nontender, non-distended; extremities: no cyanosis, no clubbing; neurologic: no focal deficits; psychiatric: flat affect. Labs: TSH: 2; WBC: 6.0; HgbA1c: 4.5. Which of the following is the most likely diagnosis? A. Delirium B. Dementia C. Depression D. Hypoglycemia E. Normal bereavement

C. Depression Ms. Martinez has a normal MoCA, so (B) is incorrect.

A 55-year-old male with a family history of melanoma presents to the clinic for evaluation of a skin lesion on his back that appeared three months ago. His wife first alerted him to it, hasn't noticed it change and he has not noticed any symptoms associated with it. Physical examination reveals a 7 mm uniformly black macule that is symmetrically round with sharply demarcated borders on his upper back near the right shoulder. Which of the following characteristics would most justify it being biopsied today? A. Borders B. Color C. Diameter D. Location E. Symmetry

C. Diameter

A 78-year-old male with a significant past medical history of chronic kidney disease stage II, coronary artery disease, and hypertension presents with lumbar back pain. He has also been feeling general malaise and chills over the past few days. On review of systems, he reports having some difficulty urinating with hesitancy and pain on urination. Currently, his chronic conditions are well managed with metoprolol, lisinopril, and aspirin. He has never smoked. Vital signs: temperature is 38 °C (100.4 °F), blood pressure is 135/75 mmHg, pulse is 76 beats/minute, and respiratory rate is 15 breaths/minute. Given this history, which of the following physical exam maneuvers would be the most helpful in making the diagnosis? A. Abdominal palpation B. Auscultation for an abdominal bruit C. Digital rectal exam D. Pinprick sensation of the legs E. Straight leg test

C. Digital rectal exam

What is the most appropriate next step? Choose the single best answer. A. Do a COVID-19 test alone. B. Do a COVID-19 test with RADT strep testing. C. Do a rapid antigen detection test for strep pharyngitis and a strep culture and COVID-19 PCR test if the strep test is negative. D. Do a rapid antigen detection test for strep pharyngitis and do a COVID-19 PCR test if the strep test is negative. Do not send a strep culture. E. Do a throat culture (sent to lab and results in two to three days) before making a treatment decision. F. Give empiric antibiotics for strep pharyngitis. G. Symptomatic treatment only. Antibiotics are not indicated in this patient.

C. Do a rapid antigen detection test for strep pharyngitis and a strep culture and COVID-19 PCR test if the strep test is negative. -a negative RADT in children over age 3 requires a confirmatory culture -(A) and (B) are incorrect as CDC guidelines for children presenting with COVID-19-like symptoms who have had no known close contact with a COVID-19 patient or PUI (person under investigation) and are not at increased likelihood for COVID-19 exposure should be tested for other disease processes first before COVID-19 testing.

A 25-year-old patient presents to the office for follow-up on anxiety and tobacco dependence. She reports she is doing well on her new medication to help with both her mood and smoking, though she continues to smoke. She recently started her first sexual relationship with a new female partner. She received a tetanus vaccine at the age of 18, and she received her flu vaccine this year. Her blood pressure is 122/70, and her physical exam is within normal limits. You review her recent Pap test, which was negative. Which of the following indicates the vaccines she should receive today? Choose the single best answer. A. HPV vaccine alone B. HPV vaccine and pneumococcal conjugate vaccine (PCV13) C. HPV vaccine and pneumococcal polysaccharide vaccine (PPSV23) D. HPV vaccine and Zoster vaccine E. No vaccines are needed

C. HPV vaccine and pneumococcal polysaccharide vaccine (PPSV23) The pneumococcal polysaccharide vaccine (PPSV23) is recommended for adults who smoke (like this patient); have chronic heart, lung, or liver illness; have alcohol use disorder; or have diabetes. The pneumococcal conjugate vaccine (PCV13) is not routinely recommended for adults, though it can be given to some patients at the age of 65. The HPV vaccine is recommended for all persons through age 26, with the series starting between ages 9 and 11. A shared-decision making approach is recommended for adults ages 27-45. The Zoster vaccine is not recommended until age 5

A 52-year-old female with a history of diabetes and rheumatoid arthritis presents for a health maintenance visit. She works in an office 10 hours a day and rarely gets exercise. Her BMI is 23 and her blood pressure is 152/85. Her previous visit two months ago showed a blood pressure of 148/82. Her father had a history of diabetes and her maternal grandmother died of rheumatic heart disease at the age of 42. She admits to marijuana drug use in the past and is a nonsmoker. Which of the following is a major risk factor for coronary artery disease (CAD) that this patient has? A. Age B. Family history C. Hypertension D. Obesity E. Rheumatoid arthritis

C. Hypertension

A 42-year-old patient presents for a visit after recently being diagnosed with type 2 diabetes. She has made a plan to work on diet and exercise. Her A1C is found to be 8.0%. What is the best medicine to start at this time? A. Basal insulin B. GLP-1 receptor agonist C. Metformin D. SGLT2 inhibitor E. Sulfonylurea

C. Metformin

A 55-year-old male comes to the clinic for a visit. He has read about the dangers of being overweight and inquires about which category he fits into. He is 177.8 cm (70 in) and weighs 99.8 kgs (220 lbs), BMI = 31.6. Which of the following categories most accurately describes the patient based on his BMI? A. Ideal B. Morbidly (very severely) obese C. Obese D. Overweight E. Underweight

C. Obese

Mr. Rodriguez is a 32-year-old male who presents with three months of postprandial epigastric burning. He reports no relief with antacid therapy. He has no allergies and no significant past medical history. He reports no hematemesis, odynophagia, dysphagia, hoarseness, or sore throat. Physical exam is unremarkable other than epigastric tenderness. H. pylori IgG serology is positive. Which of the following treatment regimens will most likely be successful in treating Mr. Rodriquez? Choose the single best answer. A. Omeprazole daily for eight weeks B. Omeprazole twice daily for four weeks C. Omeprazole twice daily, clarithromycin 500 mg twice daily, and amoxicillin 1 g twice daily for 14 days D. Omeprazole daily, clarithromycin 500 mg twice daily, and tetracycline 500 mg three times daily for 14 days E. Omeprazole twice daily, tetracycline 500 mg three times daily, and metronidazole 500 mg twice daily for 10 days

C. Omeprazole twice daily, clarithromycin 500 mg twice daily, and amoxicillin 1 g twice daily for 14 days The eradication of H. pylori requires triple or quadruple therapy, and with rates of eradication decreasing due to increased antibiotic resistance, quadruple therapy is now recommended first-line when resistance is unknown. Options include twice-daily dosing of a proton pump inhibitor (PPI) plus amoxicillin plus clarithromycin; a PPI once or twice daily plus metronidazole, tetracycline, and bismuth subcitrate four times daily; or, for penicillin-allergic patients, twice-daily dosing of a PPI plus clarithromycin plus metronidazole. Duration of therapy should be 14 days.

A 31-year-old patient, G1P0 at 26 weeks gestation, presents to the clinic for her gestational diabetes screening. Her pregnancy has been uncomplicated. Vital signs are stable. Which of the following would require a follow up three-hour glucose tolerance test? A. Fasting serum glucose concentration of 91 mg/dL B. One-hour serum glucose concentration of 128 mg/dL C. One-hour serum glucose concentration of 158 mg/dL D. Two-hour serum glucose concentration of 134 mg/dL E. Three-hour serum glucose concentration of 136 mg/dL

C. One-hour serum glucose concentration of 158 mg/dL Screening for gestational diabetes is usually performed between 24 and 28 weeks gestation, using a one-hour glucose following a 50 g glucose load. Fasting is not absolutely required for this test. If the one-hour glucose is greater than 130 mg/dL (88-99% sensitivity) or 140 mg/dL (70-88% sensitivity), then the patient is considered to have a positive result. In the case of a positive one-hour glucose, the patient should undergo a three-hour GTT with a 100 g glucose load. A one-step screening strategy using a two-hour 75 g OGTT is also described in the American Diabetes Association guidelines, however, the American College of Obstetrics and Gynecology supports the two-step strategy. Only answer choice 'C' has a result that would require a follow up three-hour GTT.

A 53-year-old female with a past medical history of diet-controlled hypertension presents to the office with a two-month history of worsening hot flashes. Her menstrual cycles are regular, occurring every 30 to 32 days, but they have gradually lessened in duration, now lasting four to five days instead of the previous six to seven days. Vital signs and physical exam are normal. Which one of the following treatments is most likely to improve the patient's symptoms? A. Black cohosh B. Gabapentin C. Oral estrogen D. Venlafaxine E. Yoga

C. Oral estrogen

A 56-year-old female with a past medical history significant for hypothyroidism, breast cancer, and recurrent urinary tract infections presents to the office to discuss dyspareunia and hot flashes that began approximately one year ago. Her last menstrual cycle was 14 months ago, and her family history is significant for a deep vein thrombosis in her sister, diagnosed at the age of 47. She has become increasingly bothered by the frequency of her hot flashes and has stopped attending social events with her friends due to these symptoms. Her past surgical history includes a left lumpectomy. She smokes one pack per day of cigarettes and drinks alcohol only on special occasions. Which one of the following recommendations to address her menopausal symptoms would be most appropriate for this patient? A. Combined oral estrogen and progesterone daily. B. Oral estrogen daily. C. Oral venlafaxine daily. D. Oral venlafaxine daily and vaginal estrogen cream twice per week E. Vaginal estrogen cream alone.

C. Oral venlafaxine daily.

You are seeing a 33-year-old male who presents with several weeks of gradually worsening headaches that he describes as severe and upsetting. They tend to occur three to four times per week and get worse as the day goes on. He describes them as feeling "like a vice squeezing my whole head." He has tried ibuprofen for them, and it has helped some. Unfortunately, he never has ibuprofen with him at work where the headaches are more common, because his new boss does not allow medications to be kept at work without a doctor's note. He reports no nausea, vomiting, lacrimosis, or photophobia. He is not awakened from sleep by the headaches. He appears mildly distressed, and his vital signs are normal. Examination of his head and neck, fundic discs, and cranial nerves is broadly normal. What is the most appropriate step in management of his headaches? A. Instruct the patient to discontinue ibuprofen, as it is causing medication overuse headaches. B. Order a CT scan of the patient's head to rule out intracranial pathology. C. Prescribe ibuprofen to be taken as needed and provide a note for work. D. Prescribe propranolol to be taken as migraine prophylaxis. E. Prescribe sumatriptan as needed and provide a note for work.

C. Prescribe ibuprofen to be taken as needed and provide a note for work. The most appropriate first-line treatment for tension-type headaches is an NSAID, such as ibuprofen (C), on an as-needed basis.

A 60-year-old patient presents to the office with increased frequency of urination and fatigue for the past several months. She reports no fever, dysuria, back pain, diarrhea, or abdominal pain. She has noted some weight loss without working on diet or exercise. Her past medical history is significant for hyperlipidemia and hypertension, for which she takes simvastatin and lisinopril. She is a nonsmoker and consumes one to two glasses of wine per week. Her vitals are: Pulse is 70 beats/minute Blood pressure is 130/70 mmHg Body mass index is 30 kg/m2 Physical examination reveals increased pigmentation in her axilla bilaterally. Her labs are as follows: Random plasma blood glucose: 205 mg/dL Creatinine: 0.8 mg/dL TSH: 2.1 U/L. What is the next most appropriate step in establishing a diagnosis of diabetes mellitus in this patient? A. An oral glucose tolerance test B. Fasting blood glucose C. HgbA1C D. The random blood glucose is sufficient E. Urine microalbumin

D

What are the three most common causes of back pain?" Select all that apply. A. Degenerative joint disease B. Disc herniation C. Kidney stones D. Lumbar strain E. Pyelonephritis F. Spinal stenosis

D, B, A.

According to USPSTF, of the groups listed below, who should be screened for chlamydia? Select all that apply. A. All non-pregnant women B. All pregnant women C. All sexually active men D. All sexually active women 25 years or older who are at high risk E. All sexually active women under 25 years

D, E.

Based on your findings from her history and physical exam, select from the following the two top diagnoses on your differential. Choose the two best answers. She has enlarged tonsils with exudate, enlarged anterior cervical nodes, and her rash A. Allergic rhinitis/pharyngitis B. COVID-19 C. Epiglottitis D. Group A Beta-hemolytic Streptococcus (GABHS) pharyngitis E. Non-specific viral pharyngitis F. Peritonsillar abscess G. Pertussis H. Retropharyngeal abscess I. Viral croup

D, E.

Which of the following statements is most accurate about placenta previa? Choose the single best answer. A. Bleeding from a placenta previa usually occurs early in pregnancy. B. Females with placenta previa must be hospitalized due to their risk of bleeding. C. Placenta previa is more common in nulliparous than multiparous females. D. Placenta previas found at this point in pregnancy (approximately 21 weeks at the time of the ultrasound) usually resolve on their own. E. Partial or marginal placenta previa are less likely to resolve than complete placenta previa.

D.

James is brought in by his mother and father today for a well-child visit. His mother had a normal term pregnancy and uncomplicated spontaneous vaginal delivery. Thus far James has been meeting all his developmental milestones. Today James can feed himself, easily pedal a tricycle, uses three-five word sentences, can be understood by strangers 75% of the time, can copy a square, takes turns in games, and engages in fantasy play with his siblings. At what developmental age is James? A. 1 year B. 18 months C. 2 years D. 3 years E. 6 years

D. 3 years

A 64-year-old cisgender female who is overweight with well-controlled hypertension comes to your office with concerns of a lump in her breast that she noticed while showering. She reports having no pain, tenderness, or skin changes. A pertinent review of systems is negative. Menarche began at the age of 10. Her first child was born when she was 31 and she had her second and last child at the age of 33. She experienced menopause at the age of 44. Her mother died of colon cancer when she was 65 and her father passed away from metastatic prostate cancer at the age of 70. She has no history of tobacco use ever and occasionally drinks a glass of wine with dinner. Her BMI is 34. Which of the information provided thus far puts the patient at decreased risk for breast cancer? A. Age B. Age at first birth C. Age at menarche D. Age at menopause E. Weight

D. Age at menopause

A 65-year-old patient with type 2 diabetes mellitus and no other chronic health issues presents to the emergency department with altered mental status. The patient experienced no known head trauma. His vitals are: Temperature is 38.1 °C (100.6 °F) Pulse is 102 beats/minute Respiratory rate is 16 breaths/minute Blood pressure is 90/74 mmHg He responds when you say his name, and he appears well-nourished. His mucous membranes appear very dry. A neurological exam reveals no focal deficits. His plasma glucose is found to be 700 mg/dL. Urinalysis reveals no ketone bodies. What is the most likely diagnosis? A. Cardiac arrhythmia B. Cerebrovascular accident C. Diabetic ketoacidosis (DKA) D. Hyperosmolar hyperglycemic state (HHS) E. Thiamine deficiency

D. Hyperosmolar hyperglycemic state (HHS)

A 37-year-old male who drives a delivery truck presents to your clinic after acute onset of severe lower back pain that began after lifting a large package while at work. When you enter the room, you find him standing, unable to sit comfortably. On physical exam, he has limited lumbar flexion, reduced to 45 degrees, positive straight leg test at 45 degrees on the left, normal gait, but difficulty with heel walk. He has 4/5 strength on the left with ankle plantar flexion. Strength is preserved on the right. Which additional physical exam finding would be consistent with this man's level of disc herniation? A. 2/5 strength on hip flexion B. Decreased range of motion on lumbar extension C. Decreased rectal tone D. Hypoactive ankle tendon reflex E. Positive Stoop test

D. Hypoactive ankle tendon reflex

An 82-year-old presents to the office for his six-month chronic disease visit. His diabetes and hypertension are controlled on his usual home medications. He reports that his wife died four weeks ago, and he is now experiencing insomnia most days of the week and fatigue and loss of energy nearly every day; reports decreased enjoyment of his activities, such as playing chess with his neighbor; and is also experiencing loss of appetite but no weight loss. He reports no suicidal ideation and has no previous suicide attempts. Mr. Jones says he often hears his wife's voice while going to bed. He says he goes to church to pray. You are trying to determine if your patient's symptoms are normal grief or if you should diagnose and treat him for Major Depressive Disorder (MDD). Which feature of Mr. Jones' case would suggest MDD rather than a normal grief reaction? A. Change in appetite B. Fatigue C. Hearing wife's voice D. Inability to experience any pleasure E. Insomnia

D. Inability to experience any pleasure

A 57-year-old patient comes in, concerned about a 1.5 cm dark multicolored mole lateral to her left eye that has been increasing in size over the past six months. A punch biopsy shows pathology indicative of squamous cell carcinoma. What is the most appropriate intervention? A. Avoid artificial sources of UV light, such as indoor tanning B. Cryotherapy extending 4 mm beyond the lesion margins C. Excisional biopsy extending 5 mm beyond lesional border D. Mohs surgery E. Three-month follow-up visits for the first year; then every six months

D. Mohs surgery

A 24-year-old patient presents to the clinic complaining of nausea and headache for the last week. She reports no fever, changes in bowel movements, or sinus symptoms. She is sexually active with one partner and admits to not being consistent with her birth control pills. She does not remember the date of her last menstrual period and reports a history of irregular menstrual cycles. She has not taken a home pregnancy test. Her vital signs reveal a blood pressure of 124/76 mmHg, a pulse of 78/min, respirations of 20/min, temperature of 36.7 oC (98.1 oF, taken orally), and an oxygen saturation of 98% on room air. The first most appropriate step in management is to: A. Admit the patient to the hospital for monitoring. B. Ask the patient to return to the clinic after she has taken a home pregnancy test and prescribe Zofran 8mg to control her nausea. C. Obtain a serial serum hCG. D. Obtain a urine hCG. E. Perform a pelvic exam to test for sexually transmitted infections.

D. Obtain a urine hCG. serial serum hCG tests (C) are used when monitoring fetal viability, not to make the diagnosis of pregnancy.

A 36-year-old patient, G3P2 at 21 weeks gestation, returns to the clinic for an ultrasound. She currently smokes a half pack of cigarettes per day. Her last two children were delivered by cesarean section. She reports no vaginal bleeding, no urinary symptoms, and no fluid leaking. She states her baby moves "all the time." Her vitals reveal a blood pressure of 130/74 mmHg, a temperature of 36.8 oC (98.3 oF), a pulse of 82, a respiration rate of 18, and a pulse oximetry of 98%. The ultrasound reveals no fetal abnormalities, but the location of the placenta is partially covering the cervical os. The diagnosis and treatment plan is: A. Placenta previa with admittance to the hospital for fetal and maternal monitoring B. Placenta previa with immediate cesarean section C. Placenta previa with reassurance that no additional monitoring is needed D. Placenta previa with subsequent ultrasound surveillance to monitor for any progression or resolution E. Uncomplicated pregnancy

D. Placenta previa with subsequent ultrasound surveillance to monitor for any progression or resolution

A 55-year-old male with no significant past medical history presents for a routine physical exam. He last saw a doctor five years ago. Social history is remarkable for a 35-pack-year tobacco history since the age of 20. He indicates that his wife and children have urged him to quit smoking for the last few months. When you ask him if he has considered quitting, he replies, "I just don't see what the big deal is!" Which stage of change best describes this patient at this time? A. Action B. Contemplation C. Maintenance D. Precontemplation E. Preparation

D. Precontemplation

You are seeing a 28-year-old female who presents with gradually worsening headaches. She has had headaches off and on since she was a teen, similar to her mother. She typically awakens with them, and they are associated with nausea and vomiting, which can last for much of the day. The pain is throbbing and usually unilateral behind the right eye. Resting in a dark room often helps, and ibuprofen can help as well if she is able to take it at the onset of the headache. Recently, the headaches have become more common, happening two to three times per week. These have been interrupting her job as a paralegal, and she is worried she will be fired. She has tried yoga and mindfulness-based stress reduction, but these have not helped. She does not smoke, drink alcohol, or use illicit drugs. She recently stopped her oral contraceptive pills in an effort to get pregnant. Her vital signs and physical exam are perfectly normal. Of the following, which would be the most appropriate next step in managing this patient? A. Advise her to continue the ibuprofen as it has been helpful B. Discontinue the ibuprofen and prescribe acetaminophen instead C. Order a CT scan of her head D. Prescribe propranolol nightly E. Prescribe sumatriptan as needed

D. Prescribe propranolol nightly This patient is experiencing migraine headaches that are frequent enough that she is at risk of developing rebound headaches from frequent headache medication use. Given that the headaches are also interrupting her work and have not responded to behavioral strategies, it is appropriate to prescribe migraine prophylaxis, such as propranolol (D).

Samantha and Joe bring in their newly adopted 5-year-old son, Matthew, to your office with the chief concern of a sore throat, that began two days ago. He was born in the United States and was neglected by his biological parents. Samantha and Joe believe Matthew lived in an old building and ate mostly non-nutritious meals. He is behind on routine immunizations. Upon physical exam, he is playful and interactive but is frequently coughing. His vital signs are: Temperature: 37.2 °C (99.0 °F); pulse: 80; respiratory rate: 19, blood pressure: 95/63 mmHg. He has tender anterior cervical lymphadenopathy, tonsillar exudates, and palatal petechiae. Which pairing contains the most appropriate next steps in the management of this patient? A. Empiric antibiotic treatment and do NOT recommend immunizations today B. Empiric antibiotic treatment and recommend immunizations today C. Rapid strep test and do NOT recommend immunizations today D. Rapid strep test and recommend immunizations today E. Reassurance and follow-up in one week

D. Rapid strep test and recommend immunizations today

Working at your clinic, you receive a call from a patient of yours, a 45-year-old male who was seen three days ago complaining of lower back pain. At that time he had no history of trauma, pain that improved while lying down, and no neurologic deficits. He works as a truck driver. He was treated conservatively along with pharmacologic intervention with NSAIDs and muscle relaxants. He calls your office now due to only minimal improvement. And although his symptoms have not changed, he is frustrated with the slow progress, needs to get back to work as soon as possible, and is concerned this might be "something serious." Which of the following is the most appropriate next step in management? A. Ask him to double the dosage of his muscle relaxants B. Obtain a plain film x-ray C. Order an MRI D. Reassure him and schedule a follow-up appointment in a few days E. Schedule him for an appointment immediately

D. Reassure him and schedule a follow-up appointment in a few days

Ms. Smith is a 38-year-old female with a significant past medical history of hypercholesterolemia who presents to her primary care physician after four weeks of episodic epigastric discomfort. She reports a recurrent ache-like sensation in the epigastric region occurring about 10 minutes after eating and lasting for several hours. The pain does not appear to be associated with any particular food. She reports no odynophagia or dysphagia. Ms Smith is a smoker, with a 30-pack-year history. You are concerned about peptic ulcer disease. Which additional history finding would increase your level of suspicion? Choose the single best answer. A. Report of hoarseness B. Epigastric discomfort associated with recumbency or bending C. Bouts of recurrent laryngitis D. Recurrent symptoms despite consistent use of antacids E. Sour or bitter taste in mouth

D. Recurrent symptoms despite consistent use of antacids Symptoms that would increase the likelihood of PUD include pain that improves with meals (although some people experience the opposite), history of NSAID use, a pain described as "gnawing" or "hunger-like" and persistent symptoms despite adequate acid-blocking therapy (although a positive response to medication does not eliminate a diagnosis of PUD)

You are seeing a 62-year-old male with a history of osteoarthritis in his knees and well-controlled hypertension and chronic constipation. His arthritic pain has been disabling in recent months, making it very difficult for him to do his work as a plumber. He has tried full dose acetaminophen in combination with diclofenac, but he still reports 8/10 pain and stiffness. He would like to pursue other treatment options. His current medications include chlorthalidone, diclofenac, acetaminophen, and methylcellulose. On exam, he is in no acute distress and his vital signs are normal. His knees show no warmth, erythema, or gross deformity. They are stable to varus and valgus stress. The Lachman test and McMurray test are both normal. There is moderate crepitus bilaterally. Which of the following would be the most appropriate next step in the management of his pain? A. Prescribe a glucosamine sulfate and chondroitin sulfate combination pill B. Prescribe amitriptyline nightly C. Prescribe oxycodone after reviewing a pain management agreement and performing a urine drug screen D. Refer to physical therapy for strength and mobility training E. Schedule for bilateral intra-articular hyaluronic acid (viscosupplementation) injections

D. Refer to physical therapy for strength and mobility training

Ms. Jones, a 35-year-old female with a significant past medical history of SLE who had been on NSAID therapy for the previous three months, presented four weeks ago with heartburn. At that time, she reported episodic, mealtime epigastric burning radiating to the throat for the past few months. She has had no surgeries. Serologic testing for H. pylori IgG was reported to be positive a few days after her visit and she was begun on triple therapy. She now returns to the office for follow-up. Today she reports no epigastric burning or tenderness. Physical exam is not significant. Which of the following is an accepted indication for performing repeat testing at this visit for H. pylori eradication? Choose the single best answer. A. Age below 40 years B. Documentation is required for all patients with confirmed H. pylori infection C. Positive serologic test prior to therapy D. Restarting of chronic NSAID therapy for SLE E. Treatment with triple rather than quadruple therapy

D. Restarting of chronic NSAID therapy for SLE

A 41-year-old male presents to the clinic with left shoulder pain after a fall where he caught himself with his left arm. To assess the injury the patient's arms were placed in 30 degrees of horizontal forward flexion and his shoulders were abducted to 90 degrees. The patient attempted to resist downward pressure and was unable to do so. This test assesses the integrity of the: A. Biceps tendon B. Deltoid muscle C. Infraspinatus muscle D. Supraspinatus muscle E. Teres minor muscle

D. Supraspinatus muscle

You are seeing a 56-year-old male who presents with daily headaches for the past two weeks. He describes them as an intense pressure behind both eyes that is throbbing in nature. He reports that lights and loud noises bother him some, but he has been able to tolerate his usual activities as a short-haul truck driver. The headaches tend to last all day. He has not had nausea, visual changes, fever, or chills, though he has reported rhinorrhea. These symptoms have occurred in the context of him quitting tobacco, which he has found difficult. He read online about cluster headaches, and he thinks that is what he is experiencing. He would like you to prescribe oxygen treatment to help with the headaches. What aspect of this patient's story is most typical of cluster headaches? A. His pain is periorbital and bilateral. B. His photophobia and phonophobia. C. The throbbing quality of his pain. D. They are associated with rhinorrhea E. They typically last all day

D. They are associated with rhinorrhea

An otherwise healthy 57-year-old G2P2 female presents to your office with vaginal bleeding that began one week ago. She began her menses at age 13 and had regular menstrual cycles until the age of 49. She reports no tobacco or alcohol use. Further review of systems is negative. Her vital signs are normal, and her BMI is 27.8 kg/m2. Her pelvic exam reveals a small amount of dried blood in the vaginal vault and mildly atrophic vaginal mucosa. Which of the following is the most appropriate next step? A. Endometrial ablation B. Reassurance C. Referral for hysterectomy D. Transvaginal ultrasound E. Trial of oral contraceptives

D. Transvaginal ultrasound

A 27-year old patient, G2P1, returns to the clinic for her second prenatal visit. Her labs reveal that her blood type is A Rh-. She states she has done research online and is concerned that this pregnancy will result in her baby dying if it has a different blood type than her own. To reassure the patient, you explain that her team of health care providers will: A. Perform a cesarean section to prevent fetal demise B. Treat the baby with Rhogam within the first 72 hours after delivery to prevent hemolytic anemia of the newborn C. Treat the mother with penicillin during labor to prevent transmission of gram-positive bacteria D. Treat the mother with Rhogam when she is at 28-weeks gestation to prevent development of antibodies against Rh+ antigens, and if it is determined the neonate is Rh+, the mother will receive a second dose of Rhogam postpartum. E. Treat the mother with Rhogam when she is at 28-weeks gestation to prevent thalassemia in the newborn, and if it is determined the neonate is Rh+ the mother will receive a second dose of Rhogam postpartum

D. Treat the mother with Rhogam when she is at 28-weeks gestation to prevent development of antibodies against Rh+ antigens, and if it is determined the neonate is Rh+, the mother will receive a second dose of Rhogam postpartum.

Dr. Nayar asks you which of the following you recommend for Mr. Reynolds now assuming a 17.6% CV risk. Select all that apply. A. Abdominal ultrasound of aorta B. Carotid ultrasound C. Coronary Calcium Score (CAC) D. High sensitivity C-reactive protein (HS CRP) E. Low dose aspirin (81 mg daily) F. Moderate-intensity statin

E, F.

Which of the following treatments are clearly safe and probably effective for the management of hot flashes according to the best available evidence? Select all that apply. A. Bioidentical hormone replacement therapy and DHEA B. Botanicals such as black cohosh and dong quai C. Mind and body practices such as yoga, tai chi, qi gong, and acupuncture D. Phytoestrogens such as soy and red clover E. Prescription medications such as clonidine and gabapentin F. Selective serotonin reuptake inhibitors (SSRIs) and serotonin-norepinephrine reuptake inhibitors (SNRIs)

E, F. Hormone therapy still has a role in the treatment of hot flashes and other menopausal symptoms in patients at low risk for hormone-related diseases but should be used at the minimum effective dose for the least amount of time, generally 3-5 years although some women will have hot flashes off the hormones once the HT is stopped. Other prescription medications, including the antidepressants SSRIs and SNRIs, and clonidine and gabapentin, although less effective than HT for vasomotor symptoms, can be beneficial in selected patients. The National Center for Complementary and Integrative Health (NCCIH) is the Federal Government's lead agency for scientific research on the diverse medical and health care systems, practices, and products that are not generally considered part of conventional medicine. NCCIH identified some weak evidence to support the use of hypnotherapy and mindfulness for the management of menopausal symptoms, but outlines specific concerns and recommends against the use of compounded hormones marketed as bioidentical hormone replacement therapy and against the use of DHEA. Furthermore, natural medicines, such as phytoestrogens and botanicals, have not been shown to be clearly safe and effective according to usual standards for prescription medications.

If his epigastric pain does not resolve after salvage therapy, what is the next most appropriate course of action? Choose the single best answer. A. Continuation of a PPI indefinitely for symptomatic control B. Obtain a 24-hour pH probe to determine if he has GERD C. Obtain a urea breath test to verify H. pylori eradication D. Prescribe triple therapy E. Refer to a gastroenterologist for upper endoscopy/EGD with mucosal biopsy and H. pylori cultures

E. If symptoms persist despite salvage therapy, upper endoscopy/EGD is required for evaluation to rule out PUD or malignancy and to undergo mucosal biopsy for evaluation of persistent H. pylori infection. Abdominal ultrasound may also be considered to evaluate for biliary/pancreatic disease as the cause of his persistent epigastric pain.

Which of the following symptoms are most likely to be due to acute coronary syndrome? A. 23-year-old male with acute onset of difficulty breathing and hyperresonance upon lung auscultation B. 35-year-old male with chest pain radiating down his left arm after falling off a ladder at work one week ago C. 42-year-old female with a pulsating pain in the center of her chest at night D. 55-year-old female with diffuse central chest pain that is worse when lying down E. 59-year-old female with palpitations that increase with exercise and are associated with nausea and vomiting

E. 59-year-old female with palpitations that increase with exercise and are associated with nausea and vomiting Palpitations, nausea, and vomiting are seen as prodromal symptoms of ACS (E) in females more than males.

You are seeing a 32-year-old patient at 34 weeks gestation for a routine prenatal visit. Her pregnancy has been uncomplicated, and she is up to date on her immunizations and screening. She is feeling well today, reporting regular fetal movement. She reports no vaginal bleeding, leaking of fluid, and contractions. She plans to breastfeed after delivery. Her vital signs are all normal today. You engage in a discussion of postpartum contraception options with her. Which of the following would be recommended as a method of contraception? A. Combined oral contraceptive starting immediately postpartum B. Combined oral contraceptive starting six weeks postpartum C. Depo-levonorgestrel injections starting now, and every three months subsequently D. Etonogestrel/ethinyl estradiol vaginal ring (NuvaRing) starting immediately postpartum E. Levonorgestrel intrauterine device (IUD) inserted immediately after delivery of the placenta

E. Levonorgestrel intrauterine device (IUD) inserted immediately after delivery of the placenta In patients who plan to breastfeed, combined contraceptive options containing estrogen are contraindicated due to the risk of estrogen decreasing milk production. Thus, combined oral contraceptives (A and B) and the etonogestrel/ethinyl estradiol vaginal ring (D) are inappropriate for this patient. Depo-levonorgestrel is an appropriate form of contraception for breastfeeding females, however, it is not administered until after delivery (making option C incorrect). Either levonorgestrel IUDs or the copper IUD may be used in breastfeeding females. These may be inserted either immediately after delivery of the placenta or at six weeks postpartum.

Althea has no known drug allergies and her weight is 20.4 kg (45 lbs). What is the most appropriate first-line choice of antibiotics for the management of her strep pharyngitis? Choose the single best answer. A. Amoxicillin Suspension 25 mg/kg PO twice daily for 5 days B. Cephalexin 20 mg/kg/dose twice daily for 10 days C. Erythromycin Ethylsuccinate 300 mg PO twice daily for ten days (30 to 50 mg/kg/day in 2 to 4 divided doses) D. Penicillin G benzathine (Bicillin L-A) 600,000 units IM single dose (1.2 mil units IM for adults or children greater than 27 kg). E. Penicillin V 250 mg PO three times daily for 10 days

E. Penicillin V 250 mg PO three times daily for 10 days

A 47-year-old patient presents to the clinic complaining of an "itchy patch" on her skin. On further examination, you note a solid, elevated, flat-topped, 1.5 cm lesion on the extensor surface of the right forearm. How would you best describe the lesion? A. Nodule B. Macule C. Papule D. Patch E. Plaque

E. Plaque

A 15-year-old female with a history of allergic rhinitis presents to the clinic with a five-day history of productive cough and wheezing that is worse at night. She reports no shortness of breath, chest pain, or fever. The patient states that she has had similar symptoms in the past, especially when seasons change. The only reported past medical history is atopic dermatitis, which is well-controlled with an over-the-counter steroid cream. Her vitals are: temperature is 36.8 C (98.2 F), pulse is 96 beats/minute, respiratory rate is 20 breaths/minute, oxygen saturation is 98%, blood pressure is 110/72 mmHg. She is in no acute distress and is breathing comfortably. On auscultation, there are some mild wheezes on expiration but air movement is good. What is the best next step? A. Prescribe a daily corticosteroid and long-acting beta agonist inhaler B. Prescribe a daily corticosteroid inhaler C. Prescribe a long-acting beta agonist inhaler D. Prescribe antibiotic therapy E. Prescribe a short-acting beta agonist inhaler with a short course of oral steroids

E. Prescribe a short-acting beta agonist inhaler with a short course of oral steroids Because the patient is symptomatic and wheezing, she needs to be treated with a medication to control her symptoms and treat her exacerbation. A short-acting beta-agonist does just that. Oral corticosteroids could be considered if her symptoms were more severe, or if she does not improve with the short-acting beta-agonist. After controlling her symptoms and treating her exacerbation, a more in-depth interview needs to be performed around the asthma symptoms and other ancillary tests need to be ordered and interpreted including pulmonary function test. Then a decision can be made regarding daily controller inhalers. Finally, antibiotic therapy is not indicated in the treatment of an asthma exacerbation.

A 12-year-old patient is brought to the clinic with a pruritic, red, scaly rash in the creases of his elbows. He reports no new topical exposures and generally feels well. He has a history of seasonal allergies. Which of the following is the most appropriate course of action? A. Ask the patient to return to the clinic after three days B. Biopsy the skin lesion C. Prescribe oral antibiotics D. Prescribe oral corticosteroids E. Prescribe topical corticosteroids

E. Prescribe topical corticosteroids (sounds like eczema)

The Aronsen family bring their 3-week old newborn for an acute visit because his parents express worry that "something is not right." They report vomiting, which seems more than the post-feeding "dribbles" they were used to with their prior children. The baby seems more irritable and less easily consoled. They deny stool changes, fever, rash, sick contacts, or decrease in frequency of feeds. They notice that he has fewer wet diapers overall. You are concerned that, on exam, the infant does not appear as well as one week prior when he underwent his routine 2-week well baby visit. His weight has increased slightly over the past week, but not as much as expected on the growth curve. He appears to be mildly dehydrated and lethargic, but you find no other distinct physical exam signs. What is the most likely diagnosis? A. Colic B. Failure to thrive C. Gastroesophageal reflux D. Intussusception E. Pyloric stenosis

E. Pyloric stenosis Pyloric stenosis, the most common cause of nonbilious vomiting in infants, is four times more common in male infants. It presents from three weeks on, though it can present as early as the first week of life. Signs can include projectile vomiting, but parents may not note this. A firm, olive-shaped mass in the mid-epigastric region is present in 70% of cases. You would order an ultrasound to confirm the diagnosis, which has 95% sensitivity when a mass is not palpable. Colic (A) is unexplained paroxysmal bouts of fussing and crying that lasts at least three hours a day, at least three times a week, for longer than three weeks, not characterized by vomiting. Failure to thrive (B) typically presents later in life and is manifest by failure to gain weight or a visible drop off of the normal growth chart curve. Gastroesophageal reflux (C) is very common; hallmark is effortless dribbling of milk following feeds. Infant should show no sign of distress. Intussusception (D) typically presents after three months of life, and is characterized by sudden onset of severe, paroxysmal, colicky pain, recurring at frequent intervals. If not diagnosed and treated, it may develop progressive lethargy, weakness, fever, and shock.

You are on call over the weekend for the pediatric clinic and a concerned mother calls in. She has three children at home, her youngest being a 5-week-old. She's very busy and about to leave town on Monday to visit relatives out of state. She tells you that for the last week her two oldest children have had a cough, runny nose, and one or two fevers as high as 38.4 C (101.2 F), but these symptoms have improved in both children with Tylenol. Her youngest child is now having the same symptoms and has a temperature of 38.1 C (100.5 F). Because they're about to go out of town, the mother wants to know if you can call in something stronger like an antibiotic so her youngest can feel better sooner. What is the most appropriate step in managing this child? A. Call in a prescription for amoxicillin as this will cover the likely causative organisms for this infection B. Call in a prescription for a cough syrup so both child and mom can get more sleep. C. Continue to use Tylenol as needed for fevers and supportive care D. Educate mom that this is likely a viral infection and it will have to run its course E. Recommend that the infant be evaluated immediately in your office or the nearest emergency department

E. Recommend that the infant be evaluated immediately in your office or the nearest emergency department While the baby may well have the same viral URI given the likely exposure to her older siblings with similar symptoms, a 5-week-old with a fever should be evaluated for possible sepsis. Young infants are at risk of developing invasive bacterial infection due to immaturity of the immune system. Additionally, they may not manifest focal symptoms or findings other than fever. Thus, all febrile neonates (within the first three months of life) should be evaluated clinically prior to initiating treatment. It would be inappropriate to reassure the mother that the cause is a virus with only symptomatic care and Tylenol or treat with antibiotics empirically before a full evaluation.

A 68-year-old male becomes confused and agitated on post-operative day three after open cholecystectomy. He does not recall his surgery or where he is, and he wants to leave the hospital. Physical exam shows a temperature of 37.7 C (99.9 F), blood pressure of 143/89 mmHg, pulse of 90 beats/minute, respiratory rate of 13 breaths/minute, and O2 sat of 98% on room air. He appears agitated and uncomfortable. He has a Foley catheter. His physical exam is unremarkable. Labs: WBC-11,000, Urinalysis: 2+ nitrites and 2+ leukocyte esterase, 10 WBC/hpf. Glucose finger stick: 80, ECG: normal sinus rhythm at 88 beats per minute. Which of the following is the best initial step to help relieve the patient's symptoms? A. Give IV ceftriaxone B. Give oral ceftriaxone C. Order urine culture D. Order urine gram stain E. Remove urine catheter

E. Remove urine catheter

You are seeing one of your regular patients, a 65-year-old female, for a follow-up appointment for intractable knee pain from osteoarthritis. The knee pain has not responded to ibuprofen or acetaminophen. She has a past medical history that also includes obesity, diabetes, hypertension, and depression. Her current medications include aspirin, HCTZ, metformin, and duloxetine. You are considering prescribing tramadol. She has never taken any kind of opioid medication in the past. Which of the following potential problems should you inquire about when adding tramadol? A. Hypercoagulability B. Hypotension C. Rash D. Recent hyperglycemia E. Seizures

E. Seizures

An 18-year-old nonsmoking male comes to the clinic with cough, chest tightness, and difficulty breathing that has gradually worsened over the proceeding three days. His past medical history is positive only for allergic rhinitis as well as an undiagnosed chronic cough, primarily at night, since he was four years old. He has no past surgical history and no recent travel. On physical exam, you notice the patient appears in mild distress, has hunched shoulders, is using accessory muscles during respiration, and is only able to talk in two to three-word sentences. His vitals are: temperature is 36.9 C (98.4 F), pulse is 125 beats/minute, respiratory rate is 28 breaths/minute, oxygen saturation is 88%, blood pressure is 100/70 mmHg. On auscultation, you hear wheezing on inspiration and expiration throughout both lungs. You also notice a prolonged expiratory phase. What is the most likely diagnosis? A. CHF exacerbation B. Cystic Fibrosis C. Foreign body aspiration D. Pulmonary embolism E. Severe asthma exacerbation

E. Severe asthma exacerbation

A 65-year-old with a past medical history of type 2 diabetes, hypertension, and hypercholesterolemia presents with six months of insomnia despite self-medication with acetaminophen, diphenhydramine, and herbal remedies. Her height is 157 cm (62 in), and weight is 113 kg (250 lbs), BMI = 45.8 kg/m2. When considering a differential diagnosis, which one of the following is a common cause of insomnia in the elderly? A. Asymptomatic coronary artery disease B. Chronic sinusitis C. Hypoparathyroidism D. Pneumonia E. Sleep Apnea

E. Sleep Apnea

A 42-year-old male accountant with a significant past medical history of obesity presents to his primary care physician after one week of lower back pain. After moving into a new home three days ago, he woke up the next morning with bilateral lower back pain without any radiation. He reports no recent trauma, fever, chills, numbness, tingling, or incontinence. He has not had any urinary frequency or dysuria. He takes no medications and has no significant past medical history. Which additional findings in his history or physical exam would make the diagnosis of lumbosacral sprain/strain more likely? A. Abnormal gait B. Increased pain with coughing C. Loss of ankle jerk D. Point tenderness on spinous processes E. Spasm of paraspinal muscles

E. Spasm of paraspinal muscles

A 47-year-old cisgender female comes into your office for a health care maintenance exam. She has hypertension and type 2 diabetes. She is not sexually active and has not yet experienced menopause. There is no family history of cancer. Her blood pressure is 118/78, her BMI is 34, and the remainder of her physical exam is within normal limits. Her vaccinations are up to date, and she has a Pap test today and will have labs drawn. According to USPSTF, which of the following is the best recommendation to give her concerning mammography? A. Should have started at age 40 and every year thereafter B. Should have started at age 40 and every two years thereafter C. Should have started at age 45 and every year thereafter D. Start at age 50 and every year thereafter E. Start at age 50 and every two years thereafter

E. Start at age 50 and every two years thereafter

A 72-year-old patient with a 30-year history of type 2 diabetes and hypertension returns to your office for a routine visit. She is taking 20 units of insulin glargine every morning and five units of insulin aspart with meals. She is on atorvastatin 40 mg daily and lisinopril 40 mg daily. She is on no other medications. Her A1C is 6.5% and her BP today is 145/90. She notes blurry vision for the past several months and a few days of dark spots in her vision. She reports no headaches or nausea. What is the most appropriate next step to slow down the progression of diabetic retinopathy? A. Increase her insulin aspart from five units to seven units with meals. B. Increase her insulin glargine to 23 units every morning. C. Perform a fundoscopic examination and make no changes to her regimen today. D. Start her on a baby aspirin. E. Start her on a calcium channel blocker.

E. Start her on a calcium channel blocker.

Tim is a 15-year-old who comes to the clinic with his father, who is concerned Tim has Strep throat. Tim has felt tired for the past day, has a sore throat, scratchy eyes, and dry cough. On exam, his temperature is 38.6 °C (101.5 °F), pulse 75, respiratory rate 14, and blood pressure 110/65 mmHg. His pupils are equal, round, and reactive and his conjunctivae are clear. He has increased tearing in both eyes. His nares are patent with mild erythema. His oropharynx is erythematous without tonsillar enlargement or exudate. He does not have tender or swollen anterior cervical lymph nodes. What is the most appropriate course of action? A. Empiric antibiotic therapy for Strep pharyngitis B. Heterophile antibody test for infectious mononucleosis C. Rapid Antigen Detection Test for Strep pharyngitis D. Strep throat culture E. Symptomatic management without further testing

E. Symptomatic management without further testing

A 68-year-old male was diagnosed with Stage 1 essential hypertension a few months ago and has been working on diet and lifestyle modifications. He has a BMI of 28 and mild knee arthritis but no other medical diagnoses. He has been a patient of yours for several years, and returns today as planned. Today his blood pressure is 156/94 mmHg. The remainder of his cardiovascular exam is within normal limits. After counseling the patient, he agrees to start an antihypertensive medication. His creatinine is 0.9, urinalysis is normal, and electrolytes are within normal limits. Which of the following is the most appropriate class of medication to begin in this patient? A. Alpha agonist B. Beta-blocker C. Loop diuretic D. Nitrate E. Thiazide diuretic

E. Thiazide diuretic

A 25-year-old female presents with vaginal bleeding and cramping. Her last normal menstrual period was six weeks ago. The patient's vital signs are stable. On speculum exam of the vagina, there is no bleeding from the cervix. A quantitative beta-human chorionic gonadotropin (beta-hCG) level is 1492 mIU/ml. The patient is sent home and told to return to the office in 48 hours. Her beta-hCG on the return visit is 3000 mIU/ml. What is the appropriate next step in the management of this patient? A. Dilation and curettage for non-viable pregnancy B. Methotrexate for ectopic pregnancy C. Progesterone level to confirm pregnancy D. Surgery for ectopic pregnancy E. Ultrasound to confirm intrauterine pregnancy

E. Ultrasound to confirm intrauterine pregnancy Quantitative serum beta-hCG levels rise at a predictable rate, making it a useful tool to help distinguish between an intrauterine pregnancy and an ectopic pregnancy. During the first 6-7 weeks of gestation, beta-hCG levels double every 48 hours. An ultrasound will be able to detect pregnancy when beta-hCG levels > 3500 mIU/ml. An ectopic pregnancy will have beta-hCG levels lower than normal and levels aren't expected to double during early gestation. In the above case, the beta-hCG levels are doubling every 48 hours and a transvaginal ultrasound would be used to confirm intrauterine pregnancy. Progesterone levels, while still useful at some points in the workup of first-trimester vaginal bleeding, are not diagnostically useful in the setting of an hCG being available, and increasing.

A 64-year-old patient comes in for a routine physical examination. He notes that over the past few months he has had to get up to urinate in the middle of the night. Benign prostatic hypertrophy (BPH) is on your differential. What other symptom is consistent with BPH? A. Cloudy penile discharge B. Dysuria C. Erectile dysfunction D. Hematuria E. Urinary urgency

E. Urinary urgency

You are seeing a 27-year-old cisgender male with a past medical history of acne and generalized anxiety disorder who presents for a follow-up visit for one year of gradually worsening dyspnea, cough, and wheezing. At his prior visit, his physical exam revealed mild diffuse expiratory wheezing. He was prescribed an albuterol inhaler, and pulmonary function tests (PFTs) were ordered. Today he reports feeling no better. The inhaler has not seemed to improve his symptoms. He is a non-smoker, and he does not use alcohol or illicit substances. On exam, he is mildly anxious, has normal vitals, has a normal head and neck exam, and has mild diffuse wheezing. His recent PFTs reveal mild flattening of the inspiratory portion of the flow-volume loop, a normal FEV1/FVC ratio, and no reversibility of findings with the administration of albuterol. Of the following, which is the most likely diagnosis? A. Asthma, mild intermittent B. Asthma, moderate persistent C. Chronic sinusitis D. Upper airway cough syndrome E. Vocal cord dysfunction

E. Vocal cord dysfunction Vocal cord dysfunction presents with chronic cough and wheezing that is unresponsive to albuterol and does not show obstructive findings or reversibility with albuterol during pulmonary function testing. Its symptoms are very similar to asthma, and it is often initially misdiagnosed as asthma. It is more common among patients with anxiety and recent life stressors. The treatment is focused on the avoidance of irritants and behavioral speech therapy.

A 34-year-old cisgender female who has no past medical problems and is not currently taking any medications comes into your office because she noticed a tender lump in her left breast starting approximately one month ago. She is worried because she has a maternal aunt who had breast cancer that was BRCA positive, though her mother is BRCA negative. Her periods have been regular since they started at the age of 13 and occur every 32 days. She is currently menstruating. She has three children, aged 12, 9, and 4. On exam, her BMI is 32, up from 28 three years ago, and her other vital signs are stable. On breast exam, you note a mobile rubbery mass approximately 1 x 1cm that has regular borders and is tender to palpation. You appreciate no axillary adenopathy. The rest of her physical exam is unremarkable. Of the information provided, which of the following puts this patient at increased risk for breast cancer? A. Age B. Age of menarche C. Family history of cancer D. Parity history E. Weight

E. Weight

A 23-year-old epileptic male presents to the emergency department after a generalized tonic-clonic seizure. You notice that the patient is holding his right shoulder and that his arm is adducted and internally rotated. What is the best imaging modality for this type of injury? A. CT scan B. MRI C. PET scan D. Ultrasound E. X-ray

E. X-ray

What would be your next step(s) in evaluating Ms. Yang's symptoms? Select all that apply. A. Admit the patient to the hospital for stress testing first thing in the morning B. Reassure her that the palpitations are likely due to the amount of stress that she is experiencing and discuss modification of CAD risk factors C. Refer the patient to cardiology for cardiac catheterization D. Schedule the patient for a cardiac stress test

Either answer choice B or D would be appropriate next steps. Reassurance and lifestyle modification (B) are reasonable long-term recommendations. Scheduling for a cardiac stress test (D) is also reasonable, given the difficulty in accurately diagnosing heart disease and her presentation. She does not have a high enough probability of acute coronary syndrome to merit hospital admission or immediate cardiac catheterization, (A) and (C).

How effective are oral medications, such as bupropion (Wellbutrin, Zyban, Budeprion) or varenicline (Chantix), in helping smokers quit? Choose the single best answer. A. Moderately effective (Quit rate at 12 months is 3 to 5 times the placebo quit rate.) B. Not effective (Quit rate at 12 months is no higher than placebo quit rate.) C. Somewhat effective (Quit rate at 12 months is 1.5 to 3 times the placebo quit rate.) D. Very effective (Quit rate at 12 months is 5 to 10 times the placebo quit rate.)

Somewhat effective (Quit rate at 12 months is 1.5 to 3 times the placebo quit rate.)

A 28-year-old, G2P1 patient delivers a 6 lb., 7oz. baby boy at 39 weeks gestation. At one minute, the baby has blue extremities and a pink body; his arms and legs are flexed and he is moving them vigorously with prompt response to stimulation; HR is 118 bpm and he is coughing and crying vigorously as well. What would his APGAR score be at one minute? A. 6 B. 7 C. 8 D. 9 E. 10

The correct answer is D.

Which of the following are risk factors for osteoporosis? Select all that apply. A. Corticosteroid use B. Family history C. Heavy alcohol use D. Hormone therapy E. Obesity

all of the above

Which of the following are risk factors for endometrial cancer? Select all that apply. A. Diabetes mellitus B. Essential hypertension C. History of breast or colon cancer D. Hypothyroidism E. Obesity

all of the above AND HT, early menarche (age 11), and age.


Set pelajaran terkait

Wisconsin Accident and Health Insurance Exam 3

View Set

CM 1011- Kimberly Williams Exam 1

View Set

Renal Ch2 - Glomerular Physiology

View Set

Automated External Defibrillator for Infants and Children less than 8 Years of Age & Ventilation Techiques

View Set

AP Euro Fall Semester Review - Final, MCQ Unit Progress #1, MCQ Unit Progress #2, ap euro mcq3, euro 3, AP EURO Exam: French Revolution, Ap European history Unit 6 Progress check

View Set